Join our FREE webinar on May 1st to learn about managing anxiety.

G
Topic
First Poster
Last Poster
k a April Highlights and 2025 AoPS Online Class Information
jlacosta   0
Apr 2, 2025
Spring is in full swing and summer is right around the corner, what are your plans? At AoPS Online our schedule has new classes starting now through July, so be sure to keep your skills sharp and be prepared for the Fall school year! Check out the schedule of upcoming classes below.

WOOT early bird pricing is in effect, don’t miss out! If you took MathWOOT Level 2 last year, no worries, it is all new problems this year! Our Worldwide Online Olympiad Training program is for high school level competitors. AoPS designed these courses to help our top students get the deep focus they need to succeed in their specific competition goals. Check out the details at this link for all our WOOT programs in math, computer science, chemistry, and physics.

Looking for summer camps in math and language arts? Be sure to check out the video-based summer camps offered at the Virtual Campus that are 2- to 4-weeks in duration. There are middle and high school competition math camps as well as Math Beasts camps that review key topics coupled with fun explorations covering areas such as graph theory (Math Beasts Camp 6), cryptography (Math Beasts Camp 7-8), and topology (Math Beasts Camp 8-9)!

Be sure to mark your calendars for the following events:
[list][*]April 3rd (Webinar), 4pm PT/7:00pm ET, Learning with AoPS: Perspectives from a Parent, Math Camp Instructor, and University Professor
[*]April 8th (Math Jam), 4:30pm PT/7:30pm ET, 2025 MATHCOUNTS State Discussion
April 9th (Webinar), 4:00pm PT/7:00pm ET, Learn about Video-based Summer Camps at the Virtual Campus
[*]April 10th (Math Jam), 4:30pm PT/7:30pm ET, 2025 MathILy and MathILy-Er Math Jam: Multibackwards Numbers
[*]April 22nd (Webinar), 4:00pm PT/7:00pm ET, Competitive Programming at AoPS (USACO).[/list]
Our full course list for upcoming classes is below:
All classes run 7:30pm-8:45pm ET/4:30pm - 5:45pm PT unless otherwise noted.

Introductory: Grades 5-10

Prealgebra 1 Self-Paced

Prealgebra 1
Sunday, Apr 13 - Aug 10
Tuesday, May 13 - Aug 26
Thursday, May 29 - Sep 11
Sunday, Jun 15 - Oct 12
Monday, Jun 30 - Oct 20
Wednesday, Jul 16 - Oct 29

Prealgebra 2 Self-Paced

Prealgebra 2
Sunday, Apr 13 - Aug 10
Wednesday, May 7 - Aug 20
Monday, Jun 2 - Sep 22
Sunday, Jun 29 - Oct 26
Friday, Jul 25 - Nov 21

Introduction to Algebra A Self-Paced

Introduction to Algebra A
Monday, Apr 7 - Jul 28
Sunday, May 11 - Sep 14 (1:00 - 2:30 pm ET/10:00 - 11:30 am PT)
Wednesday, May 14 - Aug 27
Friday, May 30 - Sep 26
Monday, Jun 2 - Sep 22
Sunday, Jun 15 - Oct 12
Thursday, Jun 26 - Oct 9
Tuesday, Jul 15 - Oct 28

Introduction to Counting & Probability Self-Paced

Introduction to Counting & Probability
Wednesday, Apr 16 - Jul 2
Thursday, May 15 - Jul 31
Sunday, Jun 1 - Aug 24
Thursday, Jun 12 - Aug 28
Wednesday, Jul 9 - Sep 24
Sunday, Jul 27 - Oct 19

Introduction to Number Theory
Thursday, Apr 17 - Jul 3
Friday, May 9 - Aug 1
Wednesday, May 21 - Aug 6
Monday, Jun 9 - Aug 25
Sunday, Jun 15 - Sep 14
Tuesday, Jul 15 - Sep 30

Introduction to Algebra B Self-Paced

Introduction to Algebra B
Wednesday, Apr 16 - Jul 30
Tuesday, May 6 - Aug 19
Wednesday, Jun 4 - Sep 17
Sunday, Jun 22 - Oct 19
Friday, Jul 18 - Nov 14

Introduction to Geometry
Wednesday, Apr 23 - Oct 1
Sunday, May 11 - Nov 9
Tuesday, May 20 - Oct 28
Monday, Jun 16 - Dec 8
Friday, Jun 20 - Jan 9
Sunday, Jun 29 - Jan 11
Monday, Jul 14 - Jan 19

Intermediate: Grades 8-12

Intermediate Algebra
Monday, Apr 21 - Oct 13
Sunday, Jun 1 - Nov 23
Tuesday, Jun 10 - Nov 18
Wednesday, Jun 25 - Dec 10
Sunday, Jul 13 - Jan 18
Thursday, Jul 24 - Jan 22

Intermediate Counting & Probability
Wednesday, May 21 - Sep 17
Sunday, Jun 22 - Nov 2

Intermediate Number Theory
Friday, Apr 11 - Jun 27
Sunday, Jun 1 - Aug 24
Wednesday, Jun 18 - Sep 3

Precalculus
Wednesday, Apr 9 - Sep 3
Friday, May 16 - Oct 24
Sunday, Jun 1 - Nov 9
Monday, Jun 30 - Dec 8

Advanced: Grades 9-12

Olympiad Geometry
Tuesday, Jun 10 - Aug 26

Calculus
Tuesday, May 27 - Nov 11
Wednesday, Jun 25 - Dec 17

Group Theory
Thursday, Jun 12 - Sep 11

Contest Preparation: Grades 6-12

MATHCOUNTS/AMC 8 Basics
Wednesday, Apr 16 - Jul 2
Friday, May 23 - Aug 15
Monday, Jun 2 - Aug 18
Thursday, Jun 12 - Aug 28
Sunday, Jun 22 - Sep 21
Tues & Thurs, Jul 8 - Aug 14 (meets twice a week!)

MATHCOUNTS/AMC 8 Advanced
Friday, Apr 11 - Jun 27
Sunday, May 11 - Aug 10
Tuesday, May 27 - Aug 12
Wednesday, Jun 11 - Aug 27
Sunday, Jun 22 - Sep 21
Tues & Thurs, Jul 8 - Aug 14 (meets twice a week!)

AMC 10 Problem Series
Friday, May 9 - Aug 1
Sunday, Jun 1 - Aug 24
Thursday, Jun 12 - Aug 28
Tuesday, Jun 17 - Sep 2
Sunday, Jun 22 - Sep 21 (1:00 - 2:30 pm ET/10:00 - 11:30 am PT)
Monday, Jun 23 - Sep 15
Tues & Thurs, Jul 8 - Aug 14 (meets twice a week!)

AMC 10 Final Fives
Sunday, May 11 - Jun 8
Tuesday, May 27 - Jun 17
Monday, Jun 30 - Jul 21

AMC 12 Problem Series
Tuesday, May 27 - Aug 12
Thursday, Jun 12 - Aug 28
Sunday, Jun 22 - Sep 21
Wednesday, Aug 6 - Oct 22

AMC 12 Final Fives
Sunday, May 18 - Jun 15

F=ma Problem Series
Wednesday, Jun 11 - Aug 27

WOOT Programs
Visit the pages linked for full schedule details for each of these programs!


MathWOOT Level 1
MathWOOT Level 2
ChemWOOT
CodeWOOT
PhysicsWOOT

Programming

Introduction to Programming with Python
Thursday, May 22 - Aug 7
Sunday, Jun 15 - Sep 14 (1:00 - 2:30 pm ET/10:00 - 11:30 am PT)
Tuesday, Jun 17 - Sep 2
Monday, Jun 30 - Sep 22

Intermediate Programming with Python
Sunday, Jun 1 - Aug 24
Monday, Jun 30 - Sep 22

USACO Bronze Problem Series
Tuesday, May 13 - Jul 29
Sunday, Jun 22 - Sep 1

Physics

Introduction to Physics
Wednesday, May 21 - Aug 6
Sunday, Jun 15 - Sep 14
Monday, Jun 23 - Sep 15

Physics 1: Mechanics
Thursday, May 22 - Oct 30
Monday, Jun 23 - Dec 15

Relativity
Sat & Sun, Apr 26 - Apr 27 (4:00 - 7:00 pm ET/1:00 - 4:00pm PT)
Mon, Tue, Wed & Thurs, Jun 23 - Jun 26 (meets every day of the week!)
0 replies
jlacosta
Apr 2, 2025
0 replies
k i Adding contests to the Contest Collections
dcouchman   1
N Apr 5, 2023 by v_Enhance
Want to help AoPS remain a valuable Olympiad resource? Help us add contests to AoPS's Contest Collections.

Find instructions and a list of contests to add here: https://artofproblemsolving.com/community/c40244h1064480_contests_to_add
1 reply
dcouchman
Sep 9, 2019
v_Enhance
Apr 5, 2023
k i Zero tolerance
ZetaX   49
N May 4, 2019 by NoDealsHere
Source: Use your common sense! (enough is enough)
Some users don't want to learn, some other simply ignore advises.
But please follow the following guideline:


To make it short: ALWAYS USE YOUR COMMON SENSE IF POSTING!
If you don't have common sense, don't post.


More specifically:

For new threads:


a) Good, meaningful title:
The title has to say what the problem is about in best way possible.
If that title occured already, it's definitely bad. And contest names aren't good either.
That's in fact a requirement for being able to search old problems.

Examples:
Bad titles:
- "Hard"/"Medium"/"Easy" (if you find it so cool how hard/easy it is, tell it in the post and use a title that tells us the problem)
- "Number Theory" (hey guy, guess why this forum's named that way¿ and is it the only such problem on earth¿)
- "Fibonacci" (there are millions of Fibonacci problems out there, all posted and named the same...)
- "Chinese TST 2003" (does this say anything about the problem¿)
Good titles:
- "On divisors of a³+2b³+4c³-6abc"
- "Number of solutions to x²+y²=6z²"
- "Fibonacci numbers are never squares"


b) Use search function:
Before posting a "new" problem spend at least two, better five, minutes to look if this problem was posted before. If it was, don't repost it. If you have anything important to say on topic, post it in one of the older threads.
If the thread is locked cause of this, use search function.

Update (by Amir Hossein). The best way to search for two keywords in AoPS is to input
[code]+"first keyword" +"second keyword"[/code]
so that any post containing both strings "first word" and "second form".


c) Good problem statement:
Some recent really bad post was:
[quote]$lim_{n\to 1}^{+\infty}\frac{1}{n}-lnn$[/quote]
It contains no question and no answer.
If you do this, too, you are on the best way to get your thread deleted. Write everything clearly, define where your variables come from (and define the "natural" numbers if used). Additionally read your post at least twice before submitting. After you sent it, read it again and use the Edit-Button if necessary to correct errors.


For answers to already existing threads:


d) Of any interest and with content:
Don't post things that are more trivial than completely obvious. For example, if the question is to solve $x^{3}+y^{3}=z^{3}$, do not answer with "$x=y=z=0$ is a solution" only. Either you post any kind of proof or at least something unexpected (like "$x=1337, y=481, z=42$ is the smallest solution). Someone that does not see that $x=y=z=0$ is a solution of the above without your post is completely wrong here, this is an IMO-level forum.
Similar, posting "I have solved this problem" but not posting anything else is not welcome; it even looks that you just want to show off what a genius you are.

e) Well written and checked answers:
Like c) for new threads, check your solutions at least twice for mistakes. And after sending, read it again and use the Edit-Button if necessary to correct errors.



To repeat it: ALWAYS USE YOUR COMMON SENSE IF POSTING!


Everything definitely out of range of common sense will be locked or deleted (exept for new users having less than about 42 posts, they are newbies and need/get some time to learn).

The above rules will be applied from next monday (5. march of 2007).
Feel free to discuss on this here.
49 replies
ZetaX
Feb 27, 2007
NoDealsHere
May 4, 2019
3 var inequality
sqing   0
a minute ago
Source: Own
Let $ a,b,c>0 . $ Prove that
$$ \left(1 +\frac{a}{b}\right)\left(1+\frac{b}{c}\right)\left(1+\frac{c}{a}\right )\geq  \frac{8}{3}\left(1+\frac{a+b}{b+c}+ \frac{b+c}{a+b}\right)$$$$ \left(1 +\frac{a^2}{b^2}\right)\left(1+\frac{b^2}{c^2}\right)\left(1+\frac{c^2}{a^2}\right )\geq \frac{8}{3}\left( 1+\frac{a^2+bc}{b^2+ca}+\frac{b^2+ca  }{a^2+bc}\right)$$
0 replies
1 viewing
sqing
a minute ago
0 replies
another problem
kjhgyuio   0
3 minutes ago
........
0 replies
kjhgyuio
3 minutes ago
0 replies
IMO 2010 Problem 5
mavropnevma   53
N 7 minutes ago by shanelin-sigma
Each of the six boxes $B_1$, $B_2$, $B_3$, $B_4$, $B_5$, $B_6$ initially contains one coin. The following operations are allowed

Type 1) Choose a non-empty box $B_j$, $1\leq j \leq 5$, remove one coin from $B_j$ and add two coins to $B_{j+1}$;

Type 2) Choose a non-empty box $B_k$, $1\leq k \leq 4$, remove one coin from $B_k$ and swap the contents (maybe empty) of the boxes $B_{k+1}$ and $B_{k+2}$.

Determine if there exists a finite sequence of operations of the allowed types, such that the five boxes $B_1$, $B_2$, $B_3$, $B_4$, $B_5$ become empty, while box $B_6$ contains exactly $2010^{2010^{2010}}$ coins.

Proposed by Hans Zantema, Netherlands
53 replies
mavropnevma
Jul 8, 2010
shanelin-sigma
7 minutes ago
IMO ShortList 1998, geometry problem 5
nttu   32
N 30 minutes ago by lpieleanu
Source: IMO ShortList 1998, geometry problem 5
Let $ABC$ be a triangle, $H$ its orthocenter, $O$ its circumcenter, and $R$ its circumradius. Let $D$ be the reflection of the point $A$ across the line $BC$, let $E$ be the reflection of the point $B$ across the line $CA$, and let $F$ be the reflection of the point $C$ across the line $AB$. Prove that the points $D$, $E$ and $F$ are collinear if and only if $OH=2R$.
32 replies
1 viewing
nttu
Oct 14, 2004
lpieleanu
30 minutes ago
a_n < b_n for large n
tastymath75025   11
N an hour ago by torch
Source: 2017 ELMO Shortlist A1
Let $0<k<\frac{1}{2}$ be a real number and let $a_0, b_0$ be arbitrary real numbers in $(0,1)$. The sequences $(a_n)_{n\ge 0}$ and $(b_n)_{n\ge 0}$ are then defined recursively by

$$a_{n+1} = \dfrac{a_n+1}{2} \text{ and } b_{n+1} = b_n^k$$
for $n\ge 0$. Prove that $a_n<b_n$ for all sufficiently large $n$.

Proposed by Michael Ma
11 replies
tastymath75025
Jul 3, 2017
torch
an hour ago
primes,exponentials,factorials
skellyrah   4
N an hour ago by aaravdodhia
find all primes p,q such that $$ \frac{p^q+q^p-p-q}{p!-q!} $$is a prime number
4 replies
skellyrah
Yesterday at 6:31 PM
aaravdodhia
an hour ago
Special line through antipodal
Phorphyrion   9
N 2 hours ago by ihategeo_1969
Source: 2025 Israel TST Test 1 P2
Triangle $\triangle ABC$ is inscribed in circle $\Omega$. Let $I$ denote its incenter and $I_A$ its $A$-excenter. Let $N$ denote the midpoint of arc $BAC$. Line $NI_A$ meets $\Omega$ a second time at $T$. The perpendicular to $AI$ at $I$ meets sides $AC$ and $AB$ at $E$ and $F$ respectively. The circumcircle of $\triangle BFT$ meets $BI_A$ a second time at $P$, and the circumcircle of $\triangle CET$ meets $CI_A$ a second time at $Q$. Prove that $PQ$ passes through the antipodal to $A$ on $\Omega$.
9 replies
Phorphyrion
Oct 28, 2024
ihategeo_1969
2 hours ago
Triangle form by perpendicular bisector
psi241   50
N 3 hours ago by Ilikeminecraft
Source: IMO Shortlist 2018 G5
Let $ABC$ be a triangle with circumcircle $\Omega$ and incentre $I$. A line $\ell$ intersects the lines $AI$, $BI$, and $CI$ at points $D$, $E$, and $F$, respectively, distinct from the points $A$, $B$, $C$, and $I$. The perpendicular bisectors $x$, $y$, and $z$ of the segments $AD$, $BE$, and $CF$, respectively determine a triangle $\Theta$. Show that the circumcircle of the triangle $\Theta$ is tangent to $\Omega$.
50 replies
psi241
Jul 17, 2019
Ilikeminecraft
3 hours ago
Sequence with infinite primes which we see again and again and again
Assassino9931   3
N 3 hours ago by grupyorum
Source: Balkan MO Shortlist 2024 N6
Let $c$ be a positive integer. Prove that there are infinitely many primes, each of which divides at least one term of the sequence $a_1 = c$, $a_{n+1} = a_n^3 + c$.
3 replies
Assassino9931
Apr 27, 2025
grupyorum
3 hours ago
Integer roots preserved under linear function of polynomial
alifenix-   23
N 3 hours ago by Mathandski
Source: USEMO 2019/2
Let $\mathbb{Z}[x]$ denote the set of single-variable polynomials in $x$ with integer coefficients. Find all functions $\theta : \mathbb{Z}[x] \to \mathbb{Z}[x]$ (i.e. functions taking polynomials to polynomials)
such that
[list]
[*] for any polynomials $p, q \in \mathbb{Z}[x]$, $\theta(p + q) = \theta(p) + \theta(q)$;
[*] for any polynomial $p \in \mathbb{Z}[x]$, $p$ has an integer root if and only if $\theta(p)$ does.
[/list]

Carl Schildkraut
23 replies
alifenix-
May 23, 2020
Mathandski
3 hours ago
BMO 2024 SL A3
MuradSafarli   5
N 3 hours ago by Nuran2010

A3.
Find all triples \((a, b, c)\) of positive real numbers that satisfy the system:
\[
\begin{aligned}
11bc - 36b - 15c &= abc \\
12ca - 10c - 28a &= abc \\
13ab - 21a - 6b &= abc.
\end{aligned}
\]
5 replies
MuradSafarli
Apr 27, 2025
Nuran2010
3 hours ago
Cool functional equation
Rayanelba   4
N 3 hours ago by ATM_
Source: Own
Find all functions $f:\mathbb{Z}_{>0}\to \mathbb{Z}_{>0}$ that verify the following equation for all $x,y\in \mathbb{Z}_{>0}$:
$max(f^{f(y)}(x),f^{f(y)}(y))|min(x,y)$
4 replies
Rayanelba
5 hours ago
ATM_
3 hours ago
af(a)+bf(b)+2ab=x^2 for all natural a, b - show that f(a)=a
shoki   26
N 3 hours ago by MathLuis
Source: Iran TST 2011 - Day 4 - Problem 3
Suppose that $f : \mathbb{N} \rightarrow \mathbb{N}$ is a function for which the expression $af(a)+bf(b)+2ab$ for all $a,b \in \mathbb{N}$ is always a perfect square. Prove that $f(a)=a$ for all $a \in \mathbb{N}$.
26 replies
shoki
May 14, 2011
MathLuis
3 hours ago
Very easy NT
GreekIdiot   8
N 3 hours ago by vsamc
Prove that there exists no natural number $n>1$ such that $n \mid 2^n-1$.
8 replies
GreekIdiot
Yesterday at 2:49 PM
vsamc
3 hours ago
Radical Center on the Euler Line (USEMO 2020/3)
franzliszt   36
N Dec 25, 2024 by ihategeo_1969
Source: USEMO 2020/3
Let $ABC$ be an acute triangle with circumcenter $O$ and orthocenter $H$. Let $\Gamma$ denote the circumcircle of triangle $ABC$, and $N$ the midpoint of $OH$. The tangents to $\Gamma$ at $B$ and $C$, and the line through $H$ perpendicular to line $AN$, determine a triangle whose circumcircle we denote by $\omega_A$. Define $\omega_B$ and $\omega_C$ similarly.
Prove that the common chords of $\omega_A$,$\omega_B$ and $\omega_C$ are concurrent on line $OH$.

Proposed by Anant Mudgal
36 replies
franzliszt
Oct 24, 2020
ihategeo_1969
Dec 25, 2024
Radical Center on the Euler Line (USEMO 2020/3)
G H J
Source: USEMO 2020/3
The post below has been deleted. Click to close.
This post has been deleted. Click here to see post.
franzliszt
23531 posts
#1 • 15 Y
Y by anantmudgal09, NJOY, lc426, Aritra12, sk2005, icematrix2, k12byda5h, jhu08, GrantStar, Mango247, Mango247, Mango247, Rounak_iitr, Siddharth03, GeoKing
Let $ABC$ be an acute triangle with circumcenter $O$ and orthocenter $H$. Let $\Gamma$ denote the circumcircle of triangle $ABC$, and $N$ the midpoint of $OH$. The tangents to $\Gamma$ at $B$ and $C$, and the line through $H$ perpendicular to line $AN$, determine a triangle whose circumcircle we denote by $\omega_A$. Define $\omega_B$ and $\omega_C$ similarly.
Prove that the common chords of $\omega_A$,$\omega_B$ and $\omega_C$ are concurrent on line $OH$.

Proposed by Anant Mudgal
This post has been edited 1 time. Last edited by franzliszt, Oct 25, 2020, 12:25 AM
Z K Y
The post below has been deleted. Click to close.
This post has been deleted. Click here to see post.
spartacle
538 posts
#2 • 6 Y
Y by SK_pi3145, Kanep, Aryan-23, Nathanisme, Ya_pank, jhu08
Complex numbers? No. Linearity? No. Barycentric coordinates? Nope. All three? Oh yes.

Bash outline
Z K Y
The post below has been deleted. Click to close.
This post has been deleted. Click here to see post.
kevinmathz
4680 posts
#3 • 52 Y
Y by mathicorn, OlympusHero, blacksheep2003, ASweatyAsianBoie, Bowser498, fidgetboss_4000, Lcz, awin, jeteagle, Inconsistent, fukano_2, mira74, Wizard_32, Zorger74, IAmTheHazard, Mathlete12345654, mlgjeffdoge21, mathiscool12, Math-wiz, azduncan, Pluto1708, Aryan-23, icematrix2, VipMath, k12byda5h, kankan, NMN12, yofro, XbenX, NJOY, lc426, anonman, Han1728, Hamroldt, Aritra12, tigerzhang, PIartist, Hypatia_K, 554183, HamstPan38825, TETris5, math31415926535, jhu08, samrocksnature, mannshah1211, rayfish, rama1728, IMUKAT, khina, HoRI_DA_GRe8, nineplusten, EpicBird08
Here's my solution that I submitted to USEMO, can people please grade it and tell me what I can expect to get as a score for this problem?
This post has been edited 2 times. Last edited by kevinmathz, Oct 25, 2020, 1:03 AM
Z K Y
The post below has been deleted. Click to close.
This post has been deleted. Click here to see post.
i3435
1350 posts
#4 • 1 Y
Y by icematrix2
I'm actually still working on it, but I found the desired concurrence point in ggb (yay) Dont look if you dont know what it is
Z K Y
The post below has been deleted. Click to close.
This post has been deleted. Click here to see post.
Imayormaynotknowcalculus
974 posts
#5 • 1 Y
Y by icematrix2
In terms of complex numbers, the concurrence point is $\frac{4(a+b+c)}{\sum_{\text{sym}}(\frac{a}{b}+1)}$ when setting $(ABC)$ as the unit circle.
This post has been edited 1 time. Last edited by Imayormaynotknowcalculus, Oct 24, 2020, 11:02 PM
Z K Y
The post below has been deleted. Click to close.
This post has been deleted. Click here to see post.
jj_ca888
2726 posts
#6 • 3 Y
Y by spartacle, SK_pi3145, icematrix2
sketch

Let $A', B', C'$ denote the intersection of the tangents and $DEF$ be the orthic. View it in the frame of $A'B'C'$ as an incenter configuration.

the main claims: radax of $\omega_A, (ABC)$ is $EF$, and cyclically.

radax of $\omega_B, \omega_C$ passes $A'$, and cyclically.

Then combine using radax spam to finish. The intersection is $X_{57}$ (from the frame $A'B'C'$).
This post has been edited 3 times. Last edited by jj_ca888, Oct 24, 2020, 11:10 PM
Z K Y
The post below has been deleted. Click to close.
This post has been deleted. Click here to see post.
SenorIncongito
307 posts
#7 • 1 Y
Y by icematrix2
kevinmathz wrote:
My solution that I submitted to USEMO

It's decept not desept lol
Z K Y
The post below has been deleted. Click to close.
This post has been deleted. Click here to see post.
mira74
1010 posts
#8 • 7 Y
Y by spartacle, blacksheep2003, SK_pi3145, SnowPanda, Aryan-23, icematrix2, lrjr24
length bash outline
This post has been edited 2 times. Last edited by mira74, Dec 5, 2020, 10:32 PM
Z K Y
The post below has been deleted. Click to close.
This post has been deleted. Click here to see post.
blacksheep2003
1081 posts
#9 • 5 Y
Y by Inconsistent, icematrix2, Mango247, Mango247, Mango247
kevinmathz wrote:
My solution that I submitted to USEMO, can people please grade it and tell me what I can expect to get as a score for this problem?

You forgot the pitchforks lol. If this is like NT3, you'll get a $-1$ on this problem :trampoline:
This post has been edited 2 times. Last edited by blacksheep2003, Oct 24, 2020, 11:16 PM
Z K Y
The post below has been deleted. Click to close.
This post has been deleted. Click here to see post.
Aryan-23
558 posts
#10 • 1 Y
Y by icematrix2
Nvm.......
This post has been edited 2 times. Last edited by Aryan-23, Oct 26, 2020, 11:55 AM
Z K Y
The post below has been deleted. Click to close.
This post has been deleted. Click here to see post.
anantmudgal09
1980 posts
#11 • 26 Y
Y by Imayormaynotknowcalculus, insertionsort, Aryan-23, pad, 606234, jj_ca888, djmathman, 62861, niyu, SnowPanda, mira74, Gaussian_cyber, Aimingformygoal, srijonrick, Ankoganit, Aryan27, Nathanisme, p_square, math_pi_rate, A-Thought-Of-God, NJOY, Mathematicsislovely, Pluto1708, icematrix2, Mango247, EpicBird08
My problem :)

I'll post my original solution here, as Evan would eventually put out the official ones :D

Comments
Attachments:
Radical_centre_on_OH.pdf (128kb)
Z K Y
The post below has been deleted. Click to close.
This post has been deleted. Click here to see post.
Bowser498
1743 posts
#12 • 2 Y
Y by OlympusHero, icematrix2
kevinmathz wrote:
My solution that I submitted to USEMO, can people please grade it and tell me what I can expect to get as a score for this problem?

Claim: kevinmathz should get a $6$ for this problem.

Proof: There are six lines in this highly detailed and concise proof and each is correct, so one point each. I was thinking of giving you a free seventh point but you used bold print (sharpie?) for part of the solution and pencil (probably mechanical?) otherwise, so you get a $6$. $\blacksquare$
SenorIncongito wrote:
It's decept not desept lol

*desert ;)
This post has been edited 1 time. Last edited by Bowser498, Oct 25, 2020, 12:36 AM
Z K Y
The post below has been deleted. Click to close.
This post has been deleted. Click here to see post.
jj_ca888
2726 posts
#14 • 1 Y
Y by icematrix2
anantmudgal09 wrote:
My problem :)

I'll post my original solution here, as Evan would eventually put out the official ones :D

Comments

Just a remark, I think claim 1 is provable with a complex bash (all points are nice).
This post has been edited 1 time. Last edited by jj_ca888, Oct 25, 2020, 12:42 AM
Z K Y
The post below has been deleted. Click to close.
This post has been deleted. Click here to see post.
v_Enhance
6877 posts
#15 • 8 Y
Y by MP8148, anantmudgal09, Nathanisme, Math_olympics, v4913, icematrix2, math31415926535, Kingsbane2139
We begin by introducing several notations. The orthic triangle is denoted $DEF$ and the tangential triangle is denoted $T_a T_b T_c$. The reflections of $H$ across the sides are denoted $H_a$, $H_b$, $H_c$. We also define the crucial points $P$ and $Q$ as the poles of $\overline{H_c B}$ and $\overline{H_b C}$ with respect to $\Gamma$.
The solution, based on the independent solutions found by Anant Mudgal and Nikolai Beluhov, hinges on two central claims: that $\omega_A$ is the circumcircle of $\triangle T_a PQ$, and that $\overline{EF}$ is the radical axis of $\Gamma$ and $\omega_A$. We prove these two claims in turn.
[asy] unitsize(2.5cm); pair A = dir(75); pair B = dir(200); pair C = dir(-20);
pair T_a = 2*B*C/(B+C); pair T_b = 2*C*A/(C+A); pair T_c = 2*A*B/(A+B); pair D = foot(A, B, C); pair E = foot(B, C, A); pair F = foot(C, A, B); pair H = A+B+C; pair H_a = -B*C/A; pair H_b = -C*A/B; pair H_c = -A*B/C;
pair Q = 2*C*H_b/(H_b+C); pair P = 2*B*H_c/(H_c+B);
filldraw(T_a--T_b--T_c--cycle, invisible, magenta); draw(A--B--C--cycle, blue); draw(A--H_a, lightblue); draw(B--H_b, lightblue); draw(C--H_c, lightblue); filldraw(unitcircle, invisible, blue);
draw(H_b--Q, red); draw(H_c--P, red); draw(Q--P, dashed+red); pair O = origin; pair N = midpoint(O--H); draw(A--N, red);
filldraw(circumcircle(T_a, P, Q), invisible, deepgreen); pair R1 = IP(unitcircle, circumcircle(T_a, P, Q)); pair R2 = OP(unitcircle, circumcircle(T_a, P, Q)); draw(R1--R2, deepgreen+1);
dot("$A$", A, dir(A)); dot("$B$", B, dir(B)); dot("$C$", C, dir(C)); dot("$T_a$", T_a, dir(T_a)); dot("$T_b$", T_b, dir(T_b)); dot("$T_c$", T_c, dir(T_c)); dot("$D$", D, dir(D)); dot("$E$", E, dir(E)); dot("$F$", F, dir(F)); dot("$H$", H, dir(H)); dot("$H_a$", H_a, dir(H_a)); dot("$H_b$", H_b, dir(H_b)); dot("$H_c$", H_c, dir(H_c)); dot("$Q$", Q, dir(Q)); dot("$P$", P, dir(P)); dot("$O$", O, dir(270)); dot("$N$", N, dir(330)); dot(R1); dot(R2);
/* TSQ Source:
!unitsize(2.5cm); A = dir 75 B = dir 200 C = dir -20
T_a = 2*B*C/(B+C) T_b = 2*C*A/(C+A) T_c = 2*A*B/(A+B) D = foot A B C E = foot B C A F = foot C A B H = A+B+C H_a = -B*C/A H_b = -C*A/B H_c = -A*B/C
Q = 2*C*H_b/(H_b+C) P = 2*B*H_c/(H_c+B)
T_a--T_b--T_c--cycle 0.1 pink / magenta A--B--C--cycle blue A--H_a lightblue B--H_b lightblue C--H_c lightblue unitcircle 0.1 lightcyan / blue
H_b--Q red H_c--P red Q--P dashed red O = origin R270 N = midpoint O--H R330 A--N red
circumcircle T_a P Q 0.02 yellow / deepgreen R1 .= IP unitcircle circumcircle T_a P Q R2 .= OP unitcircle circumcircle T_a P Q R1--R2 deepgreen+1
*/ [/asy]

Claim: [Characterization of ${\omega_A}$] Line $PQ$ passes through $H$ and is perpendicular to $\overline{AN}$.
Proof. The fact that $H$ lies on line $PQ$ is immediate by Brokard's theorem.
Showing the perpendicularity is the main part. Denote by $B'$ and $C'$ the antipodes of $B$ and $C$ on $\Gamma$. Also, define $L = \overline{H_cC'} \cap \overline{H_bB'}$ and $K = \overline{BH_c} \cap \overline{CH_b}$, as shown. [asy] 	unitsize(2.5cm); 	pair A = dir(75); 	pair B = dir(200); 	pair C = dir(-20);
pair T_a = 2*B*C/(B+C); 	pair T_b = 2*C*A/(C+A); 	pair T_c = 2*A*B/(A+B); 	pair D = foot(A, B, C); 	pair E = foot(B, C, A); 	pair F = foot(C, A, B); 	pair H = A+B+C; 	pair H_a = -B*C/A; 	pair H_b = -C*A/B; 	pair H_c = -A*B/C; 	pair Bp = -B; 	pair Cp = -C;
pair Q = 2*C*H_b/(H_b+C); 	pair P = 2*B*H_c/(H_c+B); 	pair K = extension(B, H_c, C, H_b); 	pair L = extension(Cp, H_c, Bp, H_b);
filldraw(T_a--T_b--T_c--cycle, invisible, magenta); 	draw(A--B--C--cycle, blue); 	draw(A--H_a, lightblue); 	draw(B--H_b, lightblue); 	draw(C--H_c, lightblue); 	filldraw(unitcircle, invisible, blue); 	draw(B--Bp, cyan); 	draw(C--Cp, cyan);
draw(B--K--C, lightgreen); 	draw(Cp--L--Bp, deepgreen); 	draw(H_b--Q, red); 	draw(H_c--P, red); 	draw(Q--P, dashed+red); 	pair O = origin; 	pair N = midpoint(O--H); 	draw(A--N, red); 	draw(O--K, red+dashed);
dot("$A$", A, dir(A)); 	dot("$B$", B, dir(B)); 	dot("$C$", C, dir(C)); 	dot("$T_a$", T_a, dir(T_a)); 	dot("$T_b$", T_b, dir(T_b)); 	dot("$T_c$", T_c, dir(T_c)); 	dot("$D$", D, dir(D)); 	dot("$E$", E, dir(E)); 	dot("$F$", F, dir(F)); 	dot("$H$", H, dir(H)); 	dot("$H_a$", H_a, dir(H_a)); 	dot("$H_b$", H_b, dir(H_b)); 	dot("$H_c$", H_c, dir(H_c)); 	dot("$B'$", Bp, dir(Bp)); 	dot("$C'$", Cp, dir(Cp)); 	dot("$Q$", Q, dir(Q)); 	dot("$P$", P, dir(P)); 	dot("$K$", K, dir(K)); 	dot("$L$", L, dir(135)); 	dot("$O$", O, dir(270)); 	dot("$N$", N, dir(330));
/* TSQ Source:
!unitsize(3.5cm); 	A = dir 75 	B = dir 200 	C = dir -20
T_a = 2*B*C/(B+C) 	T_b = 2*C*A/(C+A) 	T_c = 2*A*B/(A+B) 	D = foot A B C 	E = foot B C A 	F = foot C A B 	H = A+B+C 	H_a = -B*C/A 	H_b = -C*A/B 	H_c = -A*B/C 	B' = -B 	C' = -C
Q = 2*C*H_b/(H_b+C) 	P = 2*B*H_c/(H_c+B) 	K = extension B H_c C H_b 	L = extension Cp H_c Bp H_b R135
T_a--T_b--T_c--cycle 0.1 pink / magenta 	A--B--C--cycle blue 	A--H_a lightblue 	B--H_b lightblue 	C--H_c lightblue 	unitcircle 0.1 lightcyan / blue 	B--Bp cyan 	C--Cp cyan
B--K--C lightgreen 	Cp--L--Bp deepgreen 	H_b--Q red 	H_c--P red 	Q--P dashed red 	O = origin R270 	N = midpoint O--H R330 	A--N red 	O--K red dashed
*/ 	[/asy] We observe that:
  • We have $\overline{OK} \perp \overline{PQ}$ since $K$ is the pole of line $\overline{PQ}$ (again by Brokard).
  • The points $O$, $K$, $L$ are collinear by Pascal's theorem on $BH_cC'CH_bB'$.
  • The point $L$ is seen to be the reflection of $H$ across $A$, so it follows $\overline{AN} \parallel \overline{OL}$ by a $\frac{1}{2}$-factor homothety at $A$.
Putting these three observations together completes the solution. $\blacksquare$

Claim: [Radical axis of $\omega_A$ and $\Gamma$] Line $EF$ coincides with the radical axis of $\omega_A$ and $\Gamma$.
Proof. Let lines $EF$ and $T_a T_c$ meet at $Z$. It suffices to show $Z$ lies on the radical axis, and then repeat the argument on the other side.
[asy] 		unitsize(2.5cm); 		pair A = dir(75); 		pair B = dir(200); 		pair C = dir(-20);
pair T_a = 2*B*C/(B+C); 		pair T_b = 2*C*A/(C+A); 		pair T_c = 2*A*B/(A+B); 		pair D = foot(A, B, C); 		pair E = foot(B, C, A); 		pair F = foot(C, A, B); 		pair H = A+B+C; 		pair H_a = -B*C/A; 		pair H_b = -C*A/B; 		pair H_c = -A*B/C;
pair Q = 2*C*H_b/(H_b+C); 		pair P = 2*B*H_c/(H_c+B);
filldraw(T_a--T_b--T_c--cycle, invisible, magenta); 		draw(A--B--C--cycle, blue); 		draw(A--H_a, lightblue); 		draw(B--H_b, lightblue); 		draw(C--H_c, lightblue); 		filldraw(unitcircle, invisible, blue);
draw(H_b--Q, deepgreen); 		draw(H_c--P, deepgreen); 		draw(Q--P, deepgreen); 		pair O = origin;
filldraw(circumcircle(T_a, P, Q), invisible, deepgreen); 		pair X = midpoint(B--C); 		pair Y = midpoint(B--H_c); 		pair Z = extension(E, F, B, P); 		pair Z2 = extension(E, F, C, Q);
draw(B--H_c, blue); 		draw(Z--Z2, red); 		draw(X--Z, orange); 		draw(P--O--T_a, orange); 		filldraw(circumcircle(B, X, Y), invisible, lightblue);
dot("$A$", A, dir(A)); 		dot("$B$", B, dir(B)); 		dot("$C$", C, dir(C)); 		dot("$T_a$", T_a, dir(T_a)); 		dot("$T_b$", T_b, dir(T_b)); 		dot("$T_c$", T_c, dir(T_c)); 		dot("$D$", D, dir(D)); 		dot("$E$", E, dir(E)); 		dot("$F$", F, dir(F)); 		dot("$H$", H, dir(H)); 		dot(H_a); 		dot(H_b); 		dot("$H_c$", H_c, dir(H_c)); 		dot("$Q$", Q, dir(Q)); 		dot("$P$", P, dir(P)); 		dot("$O$", O, dir(20)); 		dot("$X$", X, dir(315)); 		dot("$Y$", Y, dir(200)); 		dot("$Z$", Z, dir(170)); 		dot(Z2);
/* TSQ Source:
!unitsize(2.5cm); 		A = dir 75 		B = dir 200 		C = dir -20
T_a = 2*B*C/(B+C) 		T_b = 2*C*A/(C+A) 		T_c = 2*A*B/(A+B) 		D = foot A B C 		E = foot B C A 		F = foot C A B 		H = A+B+C 		H_a .= -B*C/A 		H_b .= -C*A/B 		H_c = -A*B/C
Q = 2*C*H_b/(H_b+C) 		P = 2*B*H_c/(H_c+B)
T_a--T_b--T_c--cycle 0.1 pink / magenta 		A--B--C--cycle blue 		A--H_a lightblue 		B--H_b lightblue 		C--H_c lightblue 		unitcircle 0.1 lightcyan / blue
H_b--Q deepgreen 		H_c--P deepgreen 		Q--P deepgreen 		O = origin R20
circumcircle T_a P Q 0.02 yellow / deepgreen 		X = midpoint B--C R315 		Y = midpoint B--H_c R200 		Z = extension E F B P R170 		Z2 .= extension E F C Q
B--H_c blue 		Z--Z2 red 		X--Z orange 		P--O--T_a orange 		circumcircle B X Y 0.1 lightcyan / lightblue
*/ 	[/asy] Since $\measuredangle FBZ = \measuredangle ABZ = \measuredangle BCA = \measuredangle EFA = \measuredangle ZFB$, it follows $ZB = ZF$. We introduce two other points $X$ and $Y$ on the perpendicular bisector of $\overline{BF}$: they are the midpoints of $\overline{BC}$ and $\overline{BH_c}$.
Since $OX \cdot OT_a = OB^2 = OY \cdot OP$, it follows that $XYPT_a$ is cyclic. Then \[ ZP \cdot Z T_a = ZX \cdot ZY = ZB^2 \]with the last equality since the circumcircle of $\triangle BXY$ is tangent to $\Gamma$ (by a $\frac{1}{2}$-homothety at $B$). So the proof of the claim is complete. $\blacksquare$

Finally, we are ready to finish the problem.
Claim: Line $DT_a$ coincides with the radical axis of $\omega_B$ and $\omega_C$.
Proof. The point $D$ already coincides with the radical axis because it is the radical center of $\Gamma$, $\omega_B$ and $\omega_C$. As for the point $T$, we let the tangent to $\Gamma$ at $H_a$ meet $\overline{T_a T_c}$ at $U$ and $V$; by the first claim, these lie on $\omega_C$ and $\omega_B$ respectively. [asy] 		unitsize(2.5cm); 		pair A = dir(75); 		pair B = dir(200); 		pair C = dir(-20);
pair T_a = 2*B*C/(B+C); 		pair T_b = 2*C*A/(C+A); 		pair T_c = 2*A*B/(A+B); 		pair D = foot(A, B, C); 		pair E = foot(B, C, A); 		pair F = foot(C, A, B); 		pair H = A+B+C; 		pair H_a = -B*C/A; 		pair H_b = -C*A/B; 		pair H_c = -A*B/C;
filldraw(T_a--T_b--T_c--cycle, invisible, magenta); 		draw(A--B--C--cycle, blue); 		draw(A--H_a, lightblue); 		draw(B--H_b, lightblue); 		draw(C--H_c, lightblue); 		filldraw(unitcircle, invisible, blue);
pair U = 2*B*H_a/(B+H_a); 		pair V = 2*C*H_a/(C+H_a); 		pair U1 = extension(U, H, T_b, T_c); 		pair V1 = extension(V, H, T_b, T_c); 		draw(U--U1, dotted+deepgreen); 		draw(V--V1, dotted+deepgreen); 		filldraw(circumcircle(U, U1, T_c), invisible, deepgreen); 		filldraw(circumcircle(V, V1, T_b), invisible, deepgreen); 		draw(U--V, orange);
pair Z1 = IP(circumcircle(T_c, U, U1), circumcircle(T_b, V, V1)); 		pair Z2 = OP(circumcircle(T_c, U, U1), circumcircle(T_b, V, V1)); 		draw(Z1--T_a, red+1);
dot("$A$", A, dir(A)); 		dot("$B$", B, dir(B)); 		dot("$C$", C, dir(C)); 		dot("$T_a$", T_a, dir(T_a)); 		dot("$T_b$", T_b, dir(T_b)); 		dot("$T_c$", T_c, dir(T_c)); 		dot("$D$", D, dir(D)); 		dot("$E$", E, dir(E)); 		dot("$F$", F, dir(F)); 		dot("$H$", H, dir(180)); 		dot("$H_a$", H_a, dir(H_a)); 		dot(H_b); 		dot(H_c); 		dot("$U$", U, dir(U)); 		dot("$V$", V, dir(V)); 		dot(Z1); 		dot(Z2);
/* TSQ Source:
!unitsize(2.5cm); 		A = dir 75 		B = dir 200 		C = dir -20
T_a = 2*B*C/(B+C) 		T_b = 2*C*A/(C+A) 		T_c = 2*A*B/(A+B) 		D = foot A B C 		E = foot B C A 		F = foot C A B 		H = A+B+C R180 		H_a = -B*C/A 		H_b .= -C*A/B 		H_c .= -A*B/C
T_a--T_b--T_c--cycle 0.1 pink / magenta 		A--B--C--cycle blue 		A--H_a lightblue 		B--H_b lightblue 		C--H_c lightblue 		unitcircle 0.1 lightcyan / blue
U = 2*B*H_a/(B+H_a) 		V = 2*C*H_a/(C+H_a) 		U1 := extension U H T_b T_c 		V1 := extension V H T_b T_c 		U--U1 dotted deepgreen 		V--V1 dotted deepgreen 		circumcircle U U1 T_c 0.02 yellow / deepgreen 		circumcircle V V1 T_b 0.02 yellow / deepgreen 		U--V orange
Z1 .= IP circumcircle T_c U U1 circumcircle T_b V V1 		Z2 .= OP circumcircle T_c U U1 circumcircle T_b V V1 		Z1--T_a red+1
*/ 	[/asy] We need to show $T_aU \cdot T_a T_c = T_aV \cdot T_aT_b$.
But $UVT_bT_c$ is apparently cyclic: the sides $\overline{T_b T_c}$ and $\overline{UV}$ are reflections across a line perpendicular to $\overline{AH_a}$, while the sides $\overline{UT_c}$ and $\overline{VT_b}$ are reflections across a line perpendicular to $\overline{BC}$. So this is true. $\blacksquare$
Now since $\triangle DEF$ and $\triangle XYZ$ are homothetic (their opposite sides are parallel), and their incenters are respectively $H$ and $O$, the problem is solved.


Long remark on generalization
Z K Y
The post below has been deleted. Click to close.
This post has been deleted. Click here to see post.
VulcanForge
626 posts
#16 • 2 Y
Y by GeronimoStilton, icematrix2
Solution I submitted, except with claim 3 fixed:
[asy]
 /* Geogebra to Asymptote conversion, documentation at artofproblemsolving.com/Wiki go to User:Azjps/geogebra */
import graph; size(15cm); 
real labelscalefactor = 0.5; /* changes label-to-point distance */
pen dps = linewidth(0.7) + fontsize(10); defaultpen(dps); /* default pen style */ 
pen dotstyle = black; /* point style */ 
real xmin = -13.376797666909505, xmax = 20.078175301476524, ymin = -10.309302435620912, ymax = 12.171829741312665;  /* image dimensions */
pen zzttqq = rgb(0.6,0.2,0); 

draw((-3.4850995090586943,5.000729613009447)--(-5.48,-1.94)--(3.02,-1.66)--cycle, linewidth(2) + zzttqq); 
 /* draw figures */
draw((-3.4850995090586943,5.000729613009447)--(-5.48,-1.94), linewidth(2) + zzttqq); 
draw((-5.48,-1.94)--(3.02,-1.66), linewidth(2) + zzttqq); 
draw((3.02,-1.66)--(-3.4850995090586943,5.000729613009447), linewidth(2) + zzttqq); 
draw(circle((-1.3096654762198725,0.6184162423889894), 4.892564124214645), linewidth(2)); 
draw((3.02,-1.66)--(-6.188351862785637,0.9866591664080623), linewidth(2)); 
draw((-3.4850995090586943,5.000729613009447)--(-3.191979245089046,-3.8975641146406717), linewidth(2)); 
draw((-5.48,-1.94)--(1.34661516027427,4.727109086802955), linewidth(2)); 
draw(circle((-3.2494598798435974,1.639201589521685), 5.123833975034578), linewidth(2)); 
draw(circle((3.9721204935302747,3.858685562937984), 9.951389469440347), linewidth(2)); 
draw((10.064700133344799,11.727016116810626)--(-4.652550000551516,-3.2887810318854234), linewidth(2)); 
draw((-8.27828601019996,2.6213331257039236)--(0.9368901605471199,-5.618525482418513), linewidth(2)); 
draw((1.6493553853279797,3.1409617572717043)--(-5.107959247719024,-0.21345199184111419), linewidth(2)); 
draw((-5.716609983178886,6.129952637100622)--(-0.9839712190271103,-9.268730850962726), linewidth(2)); 
draw((-8.27828601019996,2.6213331257039236)--(-0.9839712190271103,-9.268730850962726), linewidth(2)); 
draw((-0.9839712190271103,-9.268730850962726)--(10.064700133344799,11.727016116810626), linewidth(2)); 
draw((10.064700133344799,11.727016116810626)--(-8.27828601019996,2.6213331257039236), linewidth(2)); 
draw((-0.9913917832358846,6.238636671659605)--(5.103109839452877,2.298525482418511), linewidth(2)); 
draw((0.9368901605471199,-5.618525482418513)--(-4.652550000551516,-3.2887810318854234), linewidth(2)); 
draw((-6.307449999448479,-0.591218968114576)--(-5.978807234881507,3.7628225543592846), linewidth(2)); 
 /* dots and labels */
dot((-3.4850995090586943,5.000729613009447),dotstyle); 
label("$A$", (-3.317443608078171,5.389386474373382), N * labelscalefactor); 
dot((-5.48,-1.94),dotstyle); 
label("$B$", (-5.336935142616279,-1.5454712480027382), NE * labelscalefactor); 
dot((3.02,-1.66),dotstyle); 
label("$C$", (3.1601707479874612,-1.278745950988272), NE * labelscalefactor); 
dot((-0.9839712190271103,-9.268730850962726),linewidth(4pt) + dotstyle); 
label("$X$", (-0.8407087072295467,-8.975675950548581), NE * labelscalefactor); 
dot((10.064700133344799,11.727016116810626),linewidth(4pt) + dotstyle); 
label("$Y$", (10.209339311941237,11.562171919565314), NE * labelscalefactor); 
dot((-8.27828601019996,2.6213331257039236),linewidth(4pt) + dotstyle); 
label("$Z$", (-8.11849895433858,2.912651573524768), NE * labelscalefactor); 
dot((-3.325768556618951,0.16389712823146754),linewidth(4pt) + dotstyle); 
label("$H$", (-3.1650291526413326,0.47402028653536293), S * labelscalefactor); 
dot((-1.3096654762198725,0.6184162423889894),linewidth(4pt) + dotstyle); 
label("$O$", (-1.1455376181032235,0.9312636528458764), NE * labelscalefactor); 
dot((-3.2588739008539984,-1.866833493204602),linewidth(4pt) + dotstyle); 
label("$D$", (-3.088821924922913,-1.5454712480027382), NE * labelscalefactor); 
dot((-0.9895766981723405,2.445503107517211),linewidth(4pt) + dotstyle); 
label("$E$", (-1.1455376181032235,2.6840298903695112), NE * labelscalefactor); 
dot((-4.757060209702294,0.5752781473197649),linewidth(4pt) + dotstyle); 
label("$F$", (-4.612966479291297,0.893160038986667), N * labelscalefactor); 
dot((-3.191979245089046,-3.8975641146406717),linewidth(4pt) + dotstyle); 
label("$H_{A}$", (-3.0507183110637035,-3.603066396400049), NW * labelscalefactor); 
dot((1.34661516027427,4.727109086802955),linewidth(4pt) + dotstyle); 
label("$H_{B}$", (1.483611738182239,5.046453949640497), NE * labelscalefactor); 
dot((-6.188351862785637,0.9866591664080623),linewidth(4pt) + dotstyle); 
label("$H_{C}$", (-6.022800192082053,1.2741961775787616), NE * labelscalefactor); 
dot((-4.652550000551516,-3.2887810318854234),linewidth(4pt) + dotstyle); 
label("$P$", (-4.498655637713669,-2.9934085746526975), N * labelscalefactor); 
dot((0.9368901605471199,-5.618525482418513),linewidth(4pt) + dotstyle); 
label("$Q$", (1.102575599590143,-5.3177290200644745), NE * labelscalefactor); 
dot((-2.3177170164194116,0.39115668531022846),linewidth(4pt) + dotstyle); 
label("$N$", (-2.174335192301883,0.7026419696906196), E * labelscalefactor); 
dot((5.103109839452877,2.298525482418511),linewidth(4pt) + dotstyle); 
label("$R$", (5.255869510243989,2.6078226626510923), E * labelscalefactor); 
dot((-0.9913917832358846,6.238636671659605),linewidth(4pt) + dotstyle); 
label("$S$", (-0.8407087072295467,6.532494890149667), NE * labelscalefactor); 
dot((-5.978807234881507,3.7628225543592846),linewidth(4pt) + dotstyle); 
label("$K$", (-5.832282122786005,4.055759989301052), W * labelscalefactor); 
dot((-6.307449999448479,-0.591218968114576),linewidth(4pt) + dotstyle); 
label("$L$", (-6.137111033659681,-0.28805199064882614), NE * labelscalefactor); 
dot((1.6493553853279797,3.1409617572717043),linewidth(4pt) + dotstyle); 
label("$P'$", (1.7884406490559157,3.4461021675537005), SE * labelscalefactor); 
dot((-5.107959247719024,-0.21345199184111419),linewidth(4pt) + dotstyle); 
label("$Q'$", (-5.108313459461022,-0.13563753521198832), NE * labelscalefactor); 
clip((xmin,ymin)--(xmin,ymax)--(xmax,ymax)--(xmax,ymin)--cycle); 
 /* end of picture */
[/asy]

Let the tangents at $A,B,C$ determine triangle $\Delta XYZ$ as shown. Let $H_A \in \Gamma$ be the reflection of $H$ across $BC$. Let the tangent at $H_A$ intersect $XZ, XY$ at $P,Q$.

Claim 1: $HP \perp CN$

Proof: Complex numbers with $(ABC)$ as unit cirle; we have $h_A = -\frac{bc}{a}$ and $$p=\frac{2bh_A}{b+h_A} = -2 \frac{b^2c/a}{b-bc/a} = -2\frac{bc}{a-c}$$so
\begin{align*}
h-p &=a+b+c+2 \frac{bc}{a-c} \\
&= \frac{a^2+ab+ac-ac-bc-c^2+2bc}{a-c} \\
&= \frac{a^2+ab+bc-c^2}{a-c} \\
&= \frac{(a+c)(b+a-c)}{a-c} \\
\end{align*}and since $n-c = \frac{a+b+c}{2}-c = \frac{1}{2}(a+b-c)$ we have $\frac{h-p}{n-c} = 2 \frac{a+c}{a-c}$ which is purely imaginary because $$\frac{a+c}{a-c} + \frac{1/a+1/c}{1/a-1/c} = 0$$
Thus if we similarly define $H_B, H_C, R,S,K,L$ as shown in the above diagram, we have by the previous claim (and its symmetric variants) that
\begin{align*}
\omega_A &= (XRL) \\
\omega_B &= (YKQ) \\
\omega_C &= (ZSP) \\
\end{align*}
Claim 2: $PQYZ$ is cyclic (and symmetric variants)

Proof: Angle chase
\begin{align*}
\angle PZY &= 180^\circ - 2 \angle ACB = 2(90^\circ - \angle ACB) \\
&= 2 \angle H_AAC = 2 \angle H_ACQ = 180^\circ - \angle PQY \\
\end{align*}
Thus by radical axis on $\omega_B, \omega_C, (PQYZ)$ we have that $X$ is on the radical axis of $\omega_B, \omega_C$.

Claim 3: If $D$ is the foot from $A$ to $BC$, then $XD$ is the radical axis of $\omega_B$ and $\omega_C$.

Proof: Note by Brianchon on $QH_APZAY$ and $PBZYCQ$ that we have $ZDQ$ and $YDP$ collinear. Let $ZQ$ intersect $\omega_B$ again at $Q'$, and $YP$ intersect $\omega_C$ again at $P'$. To show $D$ is on the radical axis of $\omega_B, \omega_C$ it suffices to show $PQP'Q'$ cyclic, which by Reim is equivalent to $P'Q' \parallel YZ$. We want $\frac{ZQ'}{YP'} = \frac{ZD}{YD}$. Compute $ZQ'=\frac{ZK \cdot ZY}{ZQ} = \frac{(ZA-AK)YZ}{ZQ}$ and $YP' = \frac{YX \cdot YZ}{YP} = \frac{YA-YS)YZ}{YP}$ so we want $$\frac{ZA-AK}{YA-AS} = \frac{ZP \cdot ZQ}{YP \cdot YQ}$$Since $YZPQ$ is cyclic we have $\angle PYQ = \angle PZQ$ so by angle chasing and repeatedly applying law of sines
\begin{align*}
\frac{ZP \cdot ZQ}{YP \cdot YQ} &= \frac{[ZPQ]}{[YPQ]} = \frac{ZP \sin(\angle ZPQ)}{YQ \sin(\angle YQP)} \\
&= \frac{(ZB+BP) \sin(2B)}{(YC+CQ)\sin(2C)} \\
&= \frac{\sin(2B)}{\sin(2C)} \cdot \frac{c \frac{\sin(C)}{\sin(2C)} + c \frac{\sin(90-B)}{\sin(C)} \frac{\sin(90-B)}{\sin(2B)}}{b \frac{\sin(B)}{\sin(180-2B)}+b \frac{\sin(90-C)}{\sin(B)} \frac{\sin(90-C)}{\sin(2C)}} \\
\end{align*}Scaling so $a = \sin(A), b=\sin(B), c=\sin(C)$, this quantity becomes
\begin{align*}
&\frac{\sin(B)\cos(B)}{\sin(C)\cos(C)} \cdot \frac{\tan(C)+\cot(B)}{\tan(B)+\cot(C)} \\
&= \frac{\cos^2(B)}{\cos^2(C)} \cdot \frac{\sin(C)\sin(B)+\cos(B)\cos(C)}{\sin(B)\sin(C)+\cos(C)\cos(B)} \\
&= \frac{\cos^2(B)}{\cos^2(C)} \\
\end{align*}Recall that we wanted to show that this was equal to $\frac{ZA-AK}{YA-AS}$, which we can compute by repeatedly applying law of sines:
\begin{align*}
\frac{ZA-AK}{YA-AS} &= \frac{c \frac{\sin(C)}{\sin(180-2C)} - b \frac{\sin(90-A)}{\sin(B)} \frac{\sin(90-A)}{\sin(2A)}}{b \frac{\sin(B)}{\sin(180-2B)} - c \frac{\sin(90-A)}{\sin(C)} \frac{\sin(90-A)}{\sin(2A)}}\\
&= \frac{\tan(C) - \cot(A)}{\tan(B)-\cot(A)}  = \frac{\tan(C)+\cot(B+C)}{\tan(B)+\cot(B+C)} \\
&= \frac{\cos^2(B)}{\cos^2(C)} \cdot \frac{\sin(C)\cos(C)+\cot(B+C)\cos^2(C)}{\sin(B)\cos(B)+\cot(B+C)\cos^2(B)} \\
\end{align*}Thus it suffices to show that $\sin(C)\cos(C) + \cot(B+C) \cos^2(C) = \sin(B)\cos(B)+\cot(B+C)\cos^2(B)$ which rearranges to $$(\cos^2(B)-\cos^2(C))(\cos(B)\cos(C)-\sin(B)\sin(C)) = (\sin(B)\cos(B)-\sin(C)\cos(C))(\sin(B)\cos(C)+\sin(C)\cos(B))$$which is true upon expansion and noting that $\cos^2(B)-\cos^2(C) = \sin^2(C)-\sin^2(B)$.
Let $\Delta DEF$ be the orthic triangle of $\Delta ABC$; then by the previous claim we have that the radical center of $\omega_A, \omega_B, \omega_C$ is the concurrence of $XD, YE, CF$. Note that $EF \parallel YZ$, say, because they're both antiparallel to $BC$ wrt $\angle BAC$. Thus the sides of $\Delta DEF$ and $\Delta XYZ$ are parallel, and the concurrency point of $XD, YE, CF$ is the center of homothety $T$ sending $\Delta DEF$ to $\Delta XYZ$. This homothety must send the incenter $H$ of $\Delta DEF$ to the incenter $O$ of $\Delta XYZ$; thus $TOH$ are collinear and we're done.
Z K Y
The post below has been deleted. Click to close.
This post has been deleted. Click here to see post.
anser
572 posts
#17 • 2 Y
Y by SnowPanda, icematrix2
Denote the intersections of $AH, BH, CH$ with $\Gamma$ by $H_A, H_B, H_C$. Let $A'=BB\cap CC, B' = CC\cap AA, C'=AA\cap BB$. Let $X_1 = BB\cap H_CH_C, Y_1 = CC\cap H_BH_B$, and analogously define $X_2, Y_2, X_3, Y_3$. We can compute with complex numbers
that $Y_1H\perp AN$. Similar holds for the other $X$'s and $Y$'s, so we have $\omega_A = (A'X_1Y_1)$, etc.

Inverted Diagram

Now invert about $\Gamma$ (we'll keep the point names the same in the image). Let $D, E, F$ be the feet from $A, B, C$ to the opposite side. By more complex numbers, we can show that $A'DX_1Y_1$ is cyclic, and so are $B'EX_2Y_2$ and $C'FX_3Y_3$. We'll prove $CH$ is the radical axis of $(A'DX_1Y_1)$ and $(B'EX_2Y_2)$ as follows.

First, $CA' \cdot CD = CB'\cdot CE$. Now for $H$, let $A_2 = DH\cap (A'DX_1Y_1)$ and $B_2 = EH\cap (B'EX_2Y_2)$. From $\angle A_2X_1A' = 90^{\circ}$, $X_1Y_2\parallel AB$, and $X_1A'\parallel CH_C$, we have $A_2, Y_2, X_1$ collinear. Similarly, $B_2, X_1, Y_2$ are collinear. So from $A_2Y_2X_1B_2\parallel AB$ and $HD\cdot HA = HE\cdot HB$, we have $HD\cdot HA_2 = HE\cdot HB_2$.

This means that in the original non-inverted picture, $(COH_C), \omega_A, \omega_B$ have a common chord, as do $(AOH_A), \omega_B, \omega_C$ and $(BOH_B), \omega_C, \omega_A$. So the radical center of $\omega_A, \omega_B, \omega_C$ has equal power to $(AOH_A), (BOH_B), (COH_C)$. To finish, note that these three circles are coaxial with radical axis $OH$.

First time using complex on an actual contest! :P
Z K Y
The post below has been deleted. Click to close.
This post has been deleted. Click here to see post.
MarkBcc168
1595 posts
#18 • 14 Y
Y by Gaussian_cyber, anantmudgal09, SnowPanda, Flash_Sloth, Nathanisme, Aryan-23, srijonrick, k12byda5h, Kagebaka, NMN12, KST2003, icematrix2, Mango247, GeoKing
Delightful configurations, though nearly impossible to solve synthetically under the exam condition. Here is my hybrid solution which the computation is really clean.

We first define some notations. As shown in the diagram below, we let $\triangle T_aT_bT_c$ as the tangential triangle of $\triangle ABC$; $H_a,H_b,H_c$ be the reflections of $H$ across the sides of $\triangle ABC$; $A_b,A_c$ be the poles of $CH_a$ and $BH_a$ w.r.t. $\omega$; and $B_a,B_c,C_a,C_b$ be defined similarly.
[asy]
size(12cm,0);
defaultpen(fontsize(10pt));

pair inv(pair P){
   return P/(abs(P)^2);
}

pair A = dir(110);
pair B = dir(215);
pair C = dir(325);
pair H = orthocenter(A,B,C);
pair T_a = inv((B+C)/2);
pair T_b = inv((A+C)/2);
pair T_c = inv((A+B)/2);
pair H_a = reflect(B,C)*H;
pair H_b = reflect(A,C)*H;
pair H_c = reflect(A,B)*H;

pair B_a = inv((C+H_b)/2);
pair C_a = inv((B+H_c)/2);
pair A_b = inv((H_a+C)/2);
pair A_c = inv((H_a+B)/2);
pair B_c = inv((A+H_b)/2);
pair C_b = inv((A+H_c)/2);

draw(circumcircle(T_a,B_a,C_a),green);
draw(circumcircle(T_b,A_b,C_b),green);
draw(circumcircle(T_c,A_c,B_c),green);
draw(A_b--A_c ^^ B_c--B_a ^^ C_a--C_b);
draw(circle((0,0),1));
draw(B_a--C_a^^C_b--A_b^^A_c--B_c,blue);
draw(A--H_a^^B--H_b^^C--H_c,red+dashed);
draw(A--B--C--cycle,red+linewidth(1));
draw(T_a--T_b--T_c--cycle,linewidth(0.8));

dot("$A$",A,dir(A));
dot("$B$",B,dir(B));
dot("$C$",C,dir(C));
dot("$H_a$",H_a,dir(270));
dot("$H_b$",H_b,dir(H_b));
dot("$H_c$",H_c,dir(135));
dot("$T_a$",T_a,dir(270));
dot("$T_b$",T_b,dir(45));
dot("$T_c$",T_c,dir(150));

dot("$B_a$",B_a,dir(0));
dot("$C_a$",C_a,dir(195));
dot("$A_b$",A_b,dir(300));
dot("$C_b$",C_b,dir(135));
dot("$A_c$",A_c,dir(225));
dot("$B_c$",B_c,dir(80));

dot("$H$",H,dir(40));
[/asy]
The first key claim is the following.

Claim: $B_a,C_a,H$ lie on a line that is perpendicular to $AN$.

Proof: Refer to the diagram below.
[asy]
import olympiad;
import cse5;
size(10cm,0);
defaultpen(fontsize(10pt));

pair inv(pair P){
   return P/(abs(P)^2);
}

pair A = dir(110);
pair B = dir(215);
pair C = dir(325);
pair H = orthocenter(A,B,C);

pair H_b = reflect(A,C)*H;
pair H_c = reflect(A,B)*H;
pair T = extension(B,C,H_b,H_c);

pair B_a = inv((C+H_b)/2);
pair C_a = inv((B+H_c)/2);
pair O_a = B+C;
pair N = H/2;

draw(circle((0,0),1));
draw(B--H_b^^C--H_c,red);
draw(A--B--C--cycle,red+linewidth(1));
draw(B--T--H_b);
draw(T--B_a,blue);

draw(H_c--C_a--B ^^ H_b--B_a--C, blue);

draw(arc(O_a,1,180,0),heavygreen);
draw(arc(A,abs(H-A),180,0,CCW),heavygreen);
draw(A--O_a,dashed);

dot("$A$",A,dir(A));
dot("$B$",B,2*dir(B));
dot("$C$",C,dir(C));

dot("$H_b$",H_b,dir(H_b));
dot("$H_c$",H_c,dir(150));

dot("$B_a$",B_a,dir(0));
dot("$C_a$",C_a,dir(135));

dot("$H$",H,dir(90));
dot("$T$",T,dir(180));
dot("$O_a$",O_a,dir(270));
dot("$N$",N,dir(15));
[/asy]
By Brokard's theorem, $B_a,C_a$ are colinear with $BH_b\cap CH_c=H$ and $BC\cap H_bH_c=T$. Meanwhile, $HT$ is the radical axis of $\odot(H_bHH_c)$ and $\odot(BHC)$, so it must be perpendicular to the line connecting these two centers, which are $A$ and the reflection $O_a$ of $O$ across $BC$, resp. The result is then evident. $\blacksquare$
Once we obtain the above claim, it's easy to bary-bash this problem w.r.t. $\triangle T_aT_bT_c$. Set $a=T_bT_c$, $b=T_cT_a$, $c=T_aT_b$. To obtain the equation of $\omega_a$ painlessly, we observe the following.

Claim: $\triangle T_aA_cA_b \sim \triangle T_aT_bT_c$.

Proof: Straightforward angle chasing. $\blacksquare$

Analogously, we get $\triangle C_bC_aT_c\sim\triangle T_aT_bT_c$; the former has semiperimeter $T_cA = s-c$. Thus,
$$T_cC_a = a\cdot \frac{s-c}{s} \implies \text{Pow}(T_c,\omega_a) = \frac{ab(s-c)}{s}.$$Analogously, $\text{Pow}(T_b,\omega_a) = \tfrac{ac(s-b)}{s}$, so
$$\omega_a: -a^2yz - b^2xz - c^2xy + (x+y+z)\left(\frac{ac(s-b)}{s}y+\frac{ab(s-c)}{s}z\right)=0.$$By noting the analogous calculations, we get
\begin{align*}
\omega_b&: -a^2yz - b^2xz - c^2xy + (x+y+z)\left(\frac{bc(s-a)}{s}x+\frac{ab(s-c)}{s}z\right)=0, \\
\omega_c&: -a^2yz - b^2xz - c^2xy + (x+y+z)\left(\frac{bc(s-a)}{s}x+\frac{ac(s-b)}{s}y\right)=0.
\end{align*}Hence the radical center is
$$\left(\frac{s}{bc(s-a)} : \frac{s}{ac(s-b)} : \frac{s}{ab(s-c)}\right) = \left(\frac{a}{s-a} : \frac{b}{s-b} : \frac{c}{s-c}\right),$$which can be easily verified lie on the $OI$-line of $\triangle T_aT_bT_c$. In fact, an ETC fan might recognize this as the $X_{57}$ of $\triangle T_aT_bT_c$.
This post has been edited 1 time. Last edited by MarkBcc168, Oct 25, 2020, 9:09 AM
Z K Y
The post below has been deleted. Click to close.
This post has been deleted. Click here to see post.
Idio-logy
206 posts
#19 • 3 Y
Y by Gaussian_cyber, Nathanisme, icematrix2
Solution
Attachments:
This post has been edited 3 times. Last edited by Idio-logy, Nov 3, 2020, 10:05 AM
Z K Y
The post below has been deleted. Click to close.
This post has been deleted. Click here to see post.
Flash_Sloth
230 posts
#20 • 3 Y
Y by Kanep, amar_04, icematrix2
My very long synthetic proof: Let $D,E,F$ be the feet on $BC$, $CA$, $AB$. Let $T_{A} = BB \cap CC$, $T_{B} = CC\cap AA$,$T_{C} = AA\cap BB$. The main claim is that $T_AD$, $T_BE$, $T_CF$ are the radical axis of $\omega_B \cap \omega_C$, $\omega_C \cap \omega_A$ and $\omega_A \cap \omega_B$ respectively. If this is true, then they intersect at the homothety center sending $\triangle DEF$ to $\triangle T_AT_BT_C$, which also sends $H$ (incenter of $DEF$) to $O$ (incenter of $T_AT_BT_C$), lies on $OH$.

Let $I$, $J$ be the intersection of $EF$ and $\odot O$.
Claim 1 : $\omega_A$ passes through $I$, $J$. (which implies that $F$ lies on $\omega_A \cap \omega_B$)
a) Let $\ell_A$ be the line through $H$ perpendicular to $AH$, $S_A= \ell_A \cap AN$, then $A,F,S_A,H,E$ are concyclic, denote it as $\omega_{AH}$

b) Let $O_A$ be the reflexion of $O$ w.r.t $BC$, let $M_B$, $M_C$ be the midpoint of $HB$ and $HC$. Then $O_AH= OH_A = R = OB = O_AB$ ($H_A$ is the reflexion of $H$ through $BC$), implying that $O_AM_B \perp BH$, similarly $O_AM_C \perp CH$. Moreover, $OO_A = AH$ implying that $O_A \in AN$. So $H,S_A,M_B,O_A,M_C$ are concyclic. Denote it as $\omega_{HO_A}$.

c) Apply radical theorem on $\omega_{AH}$, $\omega_{HO_A}$ and the nine-point circle $\odot N$, we have $FE$, $\ell_A$, $M_BM_C$ are concurrent, call the intersection point $P$.

d) We show that $PI \cdot PJ = PH^2$. Indeed $PI \cdot PJ = PO^2 -r_O^2$. From Stewart's theorem, $PO^2 +PH^2 = 2(PN^2 + NH^2)$ and $PN^2 -r_N^2 = PH \cdot PS_A= PH^2 + PH \cdot HS_A = PH^2+ \frac{1}{2}AH \cdot HD = PH^2+ M_AH \cdot HD =PH^2+  r_N^2-NH^2 $. Therefore
\[ PO^2  =  2(PN^2 + NH^2) - PH^2 = 2(PH^2 +2r_N^2) -PH^2 = PH^2 +r_O^2 \]where the last equality uses the fact $r_O =2 r_N$.

e) Draw the tangents at $F$ and $M_B$ w,r.t $\odot N$. From Pascal's theorem on $FFM_CM_BM_BE$, we have $Z= FF \cap M_BM_B$ lies on $PH$, which is $\ell_A$. Similarly $Y = EE \cap M_CM_C$ also lies on $\ell_A$. Note that $[Y,Z; H,P] = \stackrel{F} =[FF,FZ; FM_C, FE ]= [F, FZ \cap \odot N; M_C,E] =-1$ are harmonic. The homothety at center $H$ of ratio 2 sends $Y$ to $K = \ell_A \cap BB$, $Z$ to $L = \ell_A \cap CC$, which gives $PK \cdot PL = PH^2$. Therefore $K,L,I,J$ are concyclic.

f) To simplify notation, we drop the subscript and denote $T = BB \cap CC$, then $OBCT$ are concyclic, denote it by $\omega_{OT}$, let $Q= EF \cap BC$, $U = TQ \cap \omega_{OT}$. Then $QI \cdot QJ = QB \cdot QC = QU \cdot QT$, hence $I,J,U,T$ are concyclic.

g) Let $M$ be the midpoint of $BC$, $G = \omega_{AH} \cap \odot O $, $M,H,G$ are collinear and $QD \cdot QM = QG \cdot QA = QU \cdot QT $. Implying that $DU \perp UT$. Together with $OU \perp UT$ we have $O,D,U$ are collinear.

h) Let $P' = \ell \cap BC$, we show that $\triangle P'QU \sim \triangle HDU$ implying that $P',H,D,U$ are concyclic. As $\angle P'QU = \angle HDU$, it suffices to show $P'Q/HD = QU/DU = OM/MD$. Let $W= EF \cap AD$, from Menelaus theorem, $P'Q/QD = WH/WD$. Let $AO \cap BC= V$, note that $W$ is the orthocenter of $\triangle AQV$, hence $VW \parallel MH$, implying that $WH/WD = VM/VD = OM/AD$. Finally as $H$ is the orthocenter of $AQM$, we have $AD/QD = MD/HD$, combining the above equalities gives the desired similarity. Hence $P',H,D,U$ are concyclic.

i) Let $R = CL \cap HD$, easy angle chasing shows that $R,C,D,U$ are concyclic. Hence $U$ is the spiral center sending $DH$ to $CL$. Finally $\angle ULH = \angle UCD = \angle BTU$. Hence $T,U,K,L$ are concyclic.

j) Finally, $T,U,K,L,I,J$ must lie on the same circle, otherwise by radical theorem $IJ,KL,UT$ are collinear, requiring $P=Q$, which implies $H \in BC$, absurd! So $T,U,K,L,I,J$ are concyclic.

Claim 2 : $T_C$ lies on the radical axis of $\omega_A \cap \omega_B$
Indeed from (e) in the claim 1 and homothety of center $H$ with ratio $2$. We have the point $K= \ell_A \cap T_{A}T_C$ such that $KH_C$, $KB$ are tangents of $\odot O$ (where $H_C= CH \cap \odot O$). Finally let $K' = H_CH_C \cap T_CT_B$, then $\odot O$ is the inscribed quadrilateral of $KK'T_BT_A$ with touch points $AB \perp CH_C$, hence $KK'T_BT_A$ is a tangential-concyclic quadrilateral, implying that $pow_{\omega_A}(T_C) = T_C K \cdot T_C T_A = T_CK' \cap T_CT_B =pow_{\omega_B}(T_C) $ which completes the proof.

P.S. I think I have indeed proved some properties that does not really required to solve the problem.
Attachments:
This post has been edited 9 times. Last edited by Flash_Sloth, Oct 25, 2020, 7:55 PM
Z K Y
The post below has been deleted. Click to close.
This post has been deleted. Click here to see post.
mathlogician
1051 posts
#21 • 4 Y
Y by Nathanisme, Aryan-23, mijail, icematrix2
Let $\triangle DEF$ be the tangential triangle, and $\triangle A'B'C'$ be the orthic triangle. Let $H_A, H_B,H_C$ be the reflections of the orthocenter $H$ across $\overline{BC},\overline{AC},\overline{AB}$, respectively. Furthermore, suppose that $\triangle DA_1A_2, \triangle EB_1B_2, \triangle FC_1C_2$ are the triangles determined in the problem.

The main point of the problem is to characterize the points $A_1,A_2$. Redefine $A_1$ to be the pole of $\overline{BH_C}$, and define $A_2$ similarly as the pole of $\overline{CH_B}$. Let $T = \overline{H_BH_C} \cap \overline{BC}$. By Brokard's Theorem, $\overline{A_1A_2}$ passes through $\overline{HT}$. But remark that $\overline{HT}$ is the radical axis of $(H_BHH_C)$ and $(BHC)$. In particular, $\overline{A_1A_2}$ is perpendicular to $\overline{AO'}$, where $O'$ is the reflection of $O$ across $\overline{BC}$. Thus $A_1,A_2$ are the points defined in the problem.

Let $\overline{B'C'}$ meet $\overline{DF}$ and $\overline{DE}$ at points $Y,Z$ respectively. I claim that $\overline{YZ}$ is the radical axis of $\omega_A$ and $\Gamma$. Let $M$ and $N$ be the midpoint of $\overline{BC}$ and $\overline{BH_C}$, respectively. By angle chasing, note that $Y$ lies on the perpendicular bisector of $\overline{BC'}$, so $Y$ lies on line $MN$. Observe that $BNOM$ is cyclic with $\overline{YB}$ tangent to its circumcirle. Moreover, note that $OM \cdot OD = OB^2 = ON \cdot OA_1$, so $A_1NMD$ is cyclic. Now $YA_1 \cdot YD = YN \cdot YM = YB^2$, so $Y$ lies on the radical axis. Similarly $Z$ also lies on the radical axis. Results for $\omega_B$ and $\omega_C$ follow similarly.

Finally, I claim that $\overline{DA'}$ is the radical axis of $\omega_B$ and $\omega_C$. $A'$ obviously lies on the radical axis by our work above. Now note that $B_2C_2FE$ is cyclic by some angle chasing, so $DC_2 \cdot DF = DB_2 \cdot DE$, and thus $D$ lies on the radical axis. Now the concurrence point is simply Brazil 2013/6, using $\triangle DEF$ as the reference triangle.
This post has been edited 2 times. Last edited by mathlogician, Oct 25, 2020, 9:23 PM
Z K Y
The post below has been deleted. Click to close.
This post has been deleted. Click here to see post.
p_square
442 posts
#22 • 10 Y
Y by Rg230403, Aimingformygoal, anantmudgal09, Aryan-23, mijail, sotpidot, icematrix2, L567, Mango247, GeoKing
Really nice question :)

I'm presenting a different purely angle chase way of proving claim 1 of the solution in post #15. The rest of my solution was fairly similar to that solution, so I haven't bothered writing it up

Lemma:
(Ik the naming feels weird, but it'll make sense soon)
An arbitrary point $H$ on side $BX$ of $\triangle AXB$ is chosen. The tangents from $A,X$ to $(ABX)$ meet at $W$. Suppose $O$ and $O_2$ are the circumcentres of $\triangle AXB$ and $AHB$, and $K$ is the reflection of $H$ across $O_2$. Than, we have that $WH \perp KO$

[asy]
size(11cm);
defaultpen(fontsize(9pt));
import olympiad;

pair A = dir(50), B = dir(270), C = dir(175), O = (0,0);
pair HA = foot(B,A,C);
pair X = 2*foot(O,B,HA)-B;
pair H = orthocenter(A,B,C);
pair N = (O+H)/2;
pair W = extension(H, foot(H,C,N), O, (A+X)/2);
pair O2 = circumcenter(A,H,B);
pair K = 2*O2 - H;
pair L = foot(O,H,W);

draw(circumcircle(A,H,B), orange);
draw(circumcircle(A,W,X), purple);
draw(unitcircle, royalblue);

draw(A--B--X--A, royalblue);
draw(K--L, fuchsia);
draw(W--H, fuchsia);
draw(C--O2, dashed+fuchsia);
draw(X--W--A, springgreen);

dot("$A$", A);
dot("$B$", B);
dot("$X$", X);
dot("$C$", C);
dot("$O$", O);
dot("$H_A$", HA);
dot("$N$", N);
dot("$H$", H);
dot("$W$", W);
dot("$L$", L);
dot("$K$", K);
dot("$O_2$", O2);
[/asy]

Let $L$ denote the second intersection of circles $(AOXW)$ and $(AHBK)$
\begin{align*}
    \angle ALW &= \angle AXW \\
    &= \angle ABX \\
    &= \pi - \angle ALH
\end{align*}So, we can observe that $WLH$ are collinear.
Since $\angle OLW = \angle KLH = \frac{\pi}{2}$ our claim follows: $OLK \perp WLH$

Now, the magic begins :)
We pay special attention to the case when $H$ is the reflection of $X$ across the foot of altitude from $A$ to $BX$
We let the foot of altitude be $H_A$ and we let $AH_A$ intersect $(ABX)$ again at $C$. The point $H$ is just the orthocenter of $ABC$
The nine-point center of $ABC$, $N$, is the midpoint of $CO_2$
Now, by our claim we have that $HW \perp CNO_2$ [$NO_2$ is parallel to $OK$]

This is exactly the result we wanted, we have a characterization for the point where the perpendicular from $H$ to $CN$ meets the $B-$tangent
This post has been edited 1 time. Last edited by p_square, Oct 30, 2020, 6:11 AM
Z K Y
The post below has been deleted. Click to close.
This post has been deleted. Click here to see post.
Abhaysingh2003
222 posts
#23 • 1 Y
Y by icematrix2
@above Nice Magic. :)
Z K Y
The post below has been deleted. Click to close.
This post has been deleted. Click here to see post.
Aryan-23
558 posts
#24 • 7 Y
Y by Nathanisme, Kanep, Lcz, 606234, A-Thought-Of-God, icematrix2, Mango247
Define $\triangle T_AT_BT_C$ and $\triangle DEF$ as the tangential and orthic triangles of $\triangle ABC$. Let the line through $H$ perpendicular to $AN$ hit $T_AB$ at $A_B$. Define $A_C,B_A,B_C,C_A,C_B$ similarly. Set $AD \cap \odot (ABC) \equiv H_A$. Define $H_B,H_C$ similarly.

We will use complex numbers with $(ABC)$ as the unit circle.

Claim 1 : $C_BB_C$ is tangent to $\odot(ABC)$ at $H_A$.

Proof : Set $B_C=CC\cap H_A$. This gives $b_c=\tfrac{2bc}{b-a}$. We will check that $HB_C \perp BN$.
We need $\tfrac {h-b_c}{b-n}\in i\mathbb R$. We have :

$$\frac {h-b_c}{2(b-n)}= \frac {a+b+c-\tfrac{2bc}{b-a}}{b-a-c}= \frac {(b+a)(b-a-c)}{(b-a)(b-a-c)}=\frac {b+a}{b-a}$$
which is evidently imaginary as desired. $\square$


Claim 2 : The points $B_CT_CT_BC_B$ lie on a circle.

Proof : We need to verify that $\tfrac {c_b-t_b}{b_c-t_b} \cdot \tfrac {b_c-t_c}{c_b-t_c} \in \mathbb R$.

Let's compute the ratios explicitly.

$$b_c-t_c= \frac{2bc}{b-a} - \frac{2ab}{a+b}= \frac{2b}{b^2-a^2} \cdot (a^2-ab+ac+bc)$$Similarly, $c_b-t_b =  \frac{2c}{c^2-a^2} \cdot (a^2+ab-ac+bc)$.

$$c_b-t_c =\frac {2bc}{c-a} - \frac {2ab}{a+b}= 2b \cdot \left ( \frac c {c-a} - \frac a{a+b} \right)= \frac {2b(a^2+bc)}{(c+a)(b-a)}$$Similarly, $b_c-t_b = \frac {2c(a^2+bc)}{(b+a)(c-a)}$.

We need :

$$ \chi \stackrel {\text {def}}{:=} \tfrac {c_b-t_b}{b_c-t_b} \cdot \tfrac {b_c-t_c}{c_b-t_c} = 
\frac { \tfrac{2c}{c^2-a^2} \cdot (a^2+ab-ac+bc)} 
{  \tfrac{2b}{b^2-a^2} \cdot (a^2-ab+ac+bc)}  \cdot \frac { \tfrac {2c(a^2+bc)}{(b+a)(c-a)} } { \tfrac {2b(a^2+bc)}{(c+a)(b-a)} } = \frac { (a^2+ab-ac+bc) (a^2-ab+ac+bc) }{ (a^2+bc)^2 } \in \mathbb R$$
We have :

$$ \overline \chi = \frac { ( \tfrac 1{a^2} + \tfrac 1{ab}- \tfrac 1{ac}+\tfrac 1{bc}) (\tfrac 1{a^2}-\tfrac 1{ab}+\tfrac 1{ac}+\tfrac 1{bc}) }{ (\tfrac 1{a^2}+ \tfrac 1{bc})^2 } = \chi$$
So the claim holds. $\square$

Computing the circumcenter :
Next we will find the circumcenter $o_b$ of $\triangle T_BB_AB_C$.

Define $x = b_a-t_b = \frac {2ab}{b-c} - \frac {2ac}{a+c} = \frac {2a(c^2+ab)}{(b-c)(c+a)}$.
Similarly define $y= b_c-t_b = \frac {2c(a^2+bc)}{(b-a)(c+a)}$.

We get $\overline x = \frac {\tfrac 2a \cdot (\tfrac 1{c^2}+ \tfrac 1{ab})}{(\tfrac 1b- \tfrac 1c) \cdot (\tfrac 1a + \tfrac 1c)}= \frac {2(c^2+ab)}{a(a+c)(c-b)}$.

Similarly $\overline y =  \frac {2(a^2+bc)}{c(a+c)(a-b)}$.

We now compute $\overline x - \overline y$ and $\overline xy - x\overline y$.

$$ \overline x -\overline y = \frac {2(c^2+ab)}{a(a+c)(c-b)} - \frac {2(a^2+bc)}{c(a+c)(a-b)}$$$$\iff \overline x -\overline y = \frac {2}{ac(a+c)(c-b)(a-b)} \cdot \left ((c^2+ab)(ac-bc)-(a^2+bc)(ac-ab) \right)$$$$\iff \overline x -\overline y = \frac {2}{ac(a+c)(c-b)(a-b)} \cdot \left ( (a-c)(a+c)(ab-ac+bc) \right)= \frac {2(a-c)(ab-ac+bc)}{ac(c-b)(a-b)}$$
$$ \overline xy - x\overline y = \frac {2(c^2+ab)}{a(a+c)(c-b)} \cdot  \frac {2c(a^2+bc)}{(b-a)(c+a)} -\frac {2a(c^2+ab)}{(b-c)(c+a)} \cdot  \frac {2(a^2+bc)}{c(a+c)(a-b)}$$$$  \overline xy - x\overline y = \frac {4(a^2+bc)(c^2+ab)}{(a+c)^2} \cdot \frac {c^2-a^2}{ac(b-a)(c-b)}$$
Now we have :

$$ o_b - \frac {2ac}{a+c} = \frac {\tfrac {2a(c^2+ab)}{(b-c)(c+a)} \cdot \tfrac {2c(a^2+bc)}{(b-a)(c+a)} \cdot  \tfrac {2(a-c)(ab-ac+bc)}{ac(c-b)(a-b)}  } {  \tfrac {4(a^2+bc)(c^2+ab)}{(a+c)^2} \cdot \tfrac {c^2-a^2}{ac(b-a)(c-b)}  }$$$$\iff o_b - \frac {2ac}{a+c} = \frac {2ac(ab-ac+bc)}{(b-c)(b-a)(a+c)} $$$$\iff o_b = \frac {2ab^2c}{(b-a)(b-c)(a+c)}$$
Similarly, the circumcenter of $\triangle T_CC_AC_B$ , say $o_c$ is given by $o_c= \tfrac {2abc^2}{(a+b)(a-c)(b-c)}$.


Claim 3 : $T_AD \perp O_BO_C$

Compute :

$$ o_b-o_c = \frac {2ab^2c}{(a+c)(a-b)(c-b)} - \frac {2abc^2}{(a+b)(a-c)(b-c)} = \frac {2abc}{c-b} \left ( \frac b{(a+c)(a-b)} + \frac c{(a+b)(a-c)} \right) $$$$ \iff o_b-o_c = \frac {2abc(a^2b+a^2c+ab^2-2abc+ac^2-b^2c-bc^2)}{(c-b)(a^2-c^2)(a^2-b^2)}$$
Next we compute :

$$2t_a - 2d = \frac {4bc}{b+c} - (a+b+c)- \frac {bc}a$$$$\iff 2t_a-2d = \frac { 4abc - (a^2b+a^2c+ab^2+2abc+ac^2) + bc^2+bc^2 }{a(b+c)} = \frac { -(a^2b+a^2c+ab^2+ac^2-2abc-bc^2-b^2c)}{a(b+c)}$$
Then if we set $ \Omega\stackrel {\text {def}}{:=} \tfrac {o_b-o_c}{2t_a-2d} = \tfrac { 2a^2bc(b+c)}{(b-c)(a^2-c^2)(a^2-b^2)}$ , then we want $\Omega \in i\mathbb R$.

We have :

$$ \overline \Omega = \frac { \tfrac 2{a^2b^2c^2} \cdot (b+c) }  { \tfrac {c-b}{bc} \cdot \tfrac {c^2-a^2}{a^2c^2} \cdot \tfrac {b^2-a^2}{a^2b^2}}= \frac {2a^2bc(b+c)}{(c-b)(c^2-a^2)(b^2-a^2)} = -\Omega$$
Hence $\Omega \in i\mathbb R$ as desired. $\square$

Note that by Claim 2, $T_A$ lies on the radical axis of $\omega_B$ and $\omega_C$. Hence Claim 3 proves that $X_AD$ is the radical axis of $\omega_B$ and $\omega_C$.
Finally note that the radical center is $X=T_AD \cap T_BE \cap T_CF$. Since $\triangle T_AT_BT_C \sim \triangle {DEF}$, hence $X$ must also lie on the line joining the incenters of $\triangle T_AT_BT_C$ and $\triangle {DEF}$ which is $OH$. $\blacksquare$
This post has been edited 4 times. Last edited by Aryan-23, Oct 27, 2020, 3:00 PM
Z K Y
The post below has been deleted. Click to close.
This post has been deleted. Click here to see post.
icematrix2
33 posts
#25
Y by
$\qquad$
This post has been edited 1 time. Last edited by icematrix2, May 22, 2021, 2:18 AM
Z K Y
The post below has been deleted. Click to close.
This post has been deleted. Click here to see post.
v_Enhance
6877 posts
#26 • 3 Y
Y by Math_olympics, v4913, icematrix2
$(ABC)$ denotes the circumcircle of triangle $ABC$.
Z K Y
The post below has been deleted. Click to close.
This post has been deleted. Click here to see post.
serpent_121
1224 posts
#27 • 1 Y
Y by icematrix2
Is there a purely complex solution to this?
Z K Y
The post below has been deleted. Click to close.
This post has been deleted. Click here to see post.
icematrix2
33 posts
#28
Y by
$\qquad$
This post has been edited 1 time. Last edited by icematrix2, May 22, 2021, 2:18 AM
Z K Y
The post below has been deleted. Click to close.
This post has been deleted. Click here to see post.
spartacle
538 posts
#29 • 1 Y
Y by icematrix2
Hm, maybe I should have been more specific.

I was referring to the linearity of power of a point: If we let $\text{Pow}(P, \omega)$ denote the power of point $P$ with respect to the circle $\omega$, then if we let
$$f(P) = \text{Pow}(P, \omega_1) - \text{Pow}(P, \omega_2)$$then $f$ is a linear function of $P$, i.e.
$$f(tA + (1-t)B) = tf(A) + (1-t)f(B)$$for any real $t$ and points $A, B$. (Here we are treating the points as vectors.)

This is relatively simple to prove. It follows from the Pythagorean Theorem and the fact that if $\omega$ has center $O$ and radius $r$, $\text{Pow}(P, \omega) = OP^2 - r^2.$

In this particular problem, spoiler
Z K Y
The post below has been deleted. Click to close.
This post has been deleted. Click here to see post.
mira74
1010 posts
#31 • 1 Y
Y by icematrix2
mira74 wrote:
length bash outline

Imma complete this like a month later now lol

full-ish length bash
This post has been edited 1 time. Last edited by mira74, Nov 23, 2020, 11:34 PM
Z K Y
The post below has been deleted. Click to close.
This post has been deleted. Click here to see post.
shalomrav
330 posts
#32 • 1 Y
Y by icematrix2
mira74 wrote:
after which it's simple to see that for some point $Z$ on $OH$ with $OZ:HZ$ symmetric in $A,B,C$, the power of $Z$ to $\omega_A$ is symmetric in $A,B,C$.
How?
Z K Y
The post below has been deleted. Click to close.
This post has been deleted. Click here to see post.
mira74
1010 posts
#33 • 3 Y
Y by SK_pi3145, Imayormaynotknowcalculus, icematrix2
@above The important property about those values is that for some $\lambda$ symmetric in $A$, $B$, and $C$, we have that $$f(Z)=\lambda\text{pow}_{\omega_A}(H)+(1-\lambda)\text{pow}_{\omega_A}(O)$$is symmetric in $A,B,C$ (which can be seen by making the term involving $\tan(A)$ in both cancel out). Hence, for this $\lambda$, we have
$$\lambda\text{pow}_{\omega_A}(H)+(1-\lambda)\text{pow}_{\omega_A}(O)=\lambda\text{pow}_{\omega_B}(H)+(1-\lambda)\text{pow}_{\omega_B}(O)=\lambda\text{pow}_{\omega_C}(H)+(1-\lambda)\text{pow}_{\omega_C}(O).$$
We take $Z$ to be the point $\lambda H +(1-\lambda) O$.


Now, we can let $f(P)=\text{pow}_{\omega_A}(P)-\text{pow}_{\omega_B}(P)$, which is linear in $P$ (because linearity of power of a point). We have
$$f(\lambda H + (1-\lambda) O)=\lambda f(H)+(1-\lambda) f(O)$$$$=\lambda\text{pow}_{\omega_A}(H)+(1-\lambda)\text{pow}_{\omega_A}(O)-(\lambda\text{pow}_{\omega_B}(H)+(1-\lambda)\text{pow}_{\omega_B}(O))=0,$$so $\text{pow}_{\omega_A}(Z)=\text{pow}_{\omega_B}(Z)$, and similarly for $C$.
Z K Y
The post below has been deleted. Click to close.
This post has been deleted. Click here to see post.
TheUltimate123
1740 posts
#34 • 1 Y
Y by icematrix2
Here is a solution that never mentions the word ``power.'' Solved with Th3Numb3rThr33.

Let \(T_AT_BT_C\) be the tangential triangle, \(DEF\) the orthic triangle, and \(M_AM_BM_C\) the medial triangle. Denote by \(H_A\), \(H_B\), \(H_C\) the reflections of \(H\) over \(\overline{BC}\), \(\overline{CA}\), \(\overline{AB}\), and denote by \(A'\), \(B'\), \(C'\) the antipodes of \(A\), \(B\), \(C\) on \(\Gamma\). Let the line through \(H\) perpendicular to \(\overline{AN}\) intersect \(\overline{BB}\) at \(X\) and \(\overline{CC}\) at \(Y\).

[asy] size(5cm); defaultpen(fontsize(10pt)); pen pri=blue; pen pri2=lightblue+white; pen sec=fuchsia; pen tri=red; pen qua=purple+pink; pen qui=heavycyan; pen fil=invisible;

pair O,A,B,C,D,EE,F,H,TA,TB,TC,NN,HB,HC,X,Y,Z,K,Z,Bp,Cp; O=origin; A=dir(120); B=dir(195); C=dir(345); D=foot(A,B,C); EE=foot(B,C,A); F=foot(C,A,B); H=A+B+C; TA=2/(1/B+1/C); TB=2/(1/C+1/A); TC=2/(1/A+1/B); NN=H/2; HB=-A*C/B; HC=-A*B/C; X=extension(TA,C,H,H+rotate(90)*(A-NN)); Y=extension(TA,B,H,H+rotate(90)*(A-NN)); K=2A-H; Z=extension(B,HC,C,HB); Bp=-B; Cp=-C;

draw(H--O,pri2+dashed); draw(A--NN,tri+dashed); draw(HC--Y--B,sec); draw(H--A,pri2); draw(B--HB,pri2); draw(C--HC,pri2); draw(Y--H,qui); filldraw(unitcircle,fil,pri); filldraw(A--B--C--cycle,fil,pri);

dot("\(A\)",A,NW); dot("\(B\)",B,B); dot("\(C\)",C,C); dot("\(O\)",O,S); dot("\(H\)",H,S); dot("\(H_B\)",HB,NE); dot("\(H_C\)",HC,NW); dot("\(X\)",Y,W); dot("\(N\)",NN,S);     [/asy]

Claim 1: \(X\) is the pole of \(\overline{BH_C}\) in \(\Gamma\), and \(Y\) is the pole of \(\overline{CH_B}\) in \(\Gamma\).

Proof. Let \(X'\) be the pole of \(\overline{BH_C}\); we will show \(X=X'\) via \(\overline{HX'}\perp\overline{AN}\).

In complex numbers, we have \(H=a+b+c\), \(N=(a+b+c)/2\), \(H_B=-ac/b\), and \[X=\frac2{\frac1c-\frac b{ac}}=\frac{2ac}{a-b}.\]
It is easy to check that \[\frac{x-h}{n-a}=\frac{\frac{2ac}{a-b}-(a+b+c)}{\frac{b+c-a}2}=\frac{\frac{(a+b)(c+b-a)}{a-b}}{\frac{b+c-a}2}=\frac{a+b}{2(a-b)}\in\mathbb I,\]as desired. \(\blacksquare\)

Invert with respect to \(\Gamma\), so that \(X\) and \(Y\) are sent to the midpoints \(X^*\) and \(Y^*\) of \(\overline{BH_C}\), \(\overline{CH_B}\).

[asy]         size(7cm); defaultpen(fontsize(10pt)); pen pri=blue; pen pri1=lightblue; pen pri2=lightblue+white; pen sec=fuchsia; pen tri=lightred; pen qua=purple+pink; pen qui=heavycyan; pen sex=olive; pen fil=invisible; pen qfil=invisible; pen sefil=invisible;

pair O,A,B,C,D,EE,F,H,TA,TB,TC,NN,HB,HC,X,Y,Z,K,Z,Bp,Cp,L,Xs,Ys,M,LB,LC,V; O=origin; A=dir(110); B=dir(205); C=dir(335); D=foot(A,B,C); EE=foot(B,C,A); F=foot(C,A,B); H=A+B+C; TA=2/(1/B+1/C); TB=2/(1/C+1/A); TC=2/(1/A+1/B); NN=H/2; HB=-A*C/B; HC=-A*B/C; X=extension(TA,B,H,H+rotate(90)*(A-NN)); Y=extension(TA,C,H,H+rotate(90)*(A-NN)); K=2A-H; Z=extension(B,HC,C,HB); Bp=-B; Cp=-C; L=(A+K)/2; Xs=(B+HC)/2; Ys=(C+HB)/2; M=(B+C)/2; LB=(B+H)/2; LC=(C+H)/2; V=A/2;

draw(B--HC,sec); draw(C--HB,sec); filldraw(circumcircle(D,LB,LC),qfil,qua+dashed); filldraw(circumcircle(D,M,L),qfil,qua); draw(Ys--M--Xs,tri); draw(A--D,pri2); draw(B--HB,pri2); draw(C--HC,pri2); draw(A--O--H,pri1); filldraw(unitcircle,fil,pri); filldraw(A--B--C--cycle,fil,pri); filldraw(D--LB--Xs--cycle,sefil,sex); filldraw(D--LC--Ys--cycle,sefil,sex); filldraw(D--NN--V--cycle,sefil,sex);

dot("\(A\)",A,NW); dot("\(B\)",B,B); dot("\(C\)",C,C); dot("\(D\)",D,S); dot("\(M_A\)",M,S); dot("\(H_B\)",HB,dir(75)); dot("\(H_C\)",HC,dir(120)); dot("\(X^*\)",Xs,W); dot("\(Y^*\)",Ys,dir(15)); dot("\(H\)",H,dir(120)); dot("\(L_B\)",LB,N); dot("\(L_C\)",LC,dir(120)); dot("\(V\)",V,NE); dot("\(N\)",NN,dir(300)); dot("\(O\)",O,S);     [/asy]

Claim 2: The image of \(\omega_A\) is the circle \(\omega_A^*\) containing \(M_A\), \(X^*\), \(Y^*\), \(D\) and centered at the midpoint \(V\) of \(\overline{AO}\).

Proof. Evidently \(\omega_A^*\) contains \(M_A\), \(X^*\), \(Y^*\).

Let \(L_B\), \(L_C\) be the midpoints of \(\overline{HB}\), \(\overline{HC}\), so that \(D\), \(M\), \(L_B\), \(L_C\) lie on the nine-point circle. I claim \(\triangle DL_BX^*\sim\triangle DL_CY^*\sim\triangle DNV\), from which the claim will follow by the spiral similarity.

Indeed, \(\angle DL_BX^*=180^\circ-\angle DL_BM_A=180^\circ-\frac12\angle DNM_A=\angle DNV\) and \[\frac{NK}{DN}=\frac{AH}{AO}=2\cos A=\frac{HH_C}{BH}=\frac{L_BM_B}{DL_B},\]so \(\triangle DL_BX^*\sim\triangle DNV\). The rest follows symmetrically. \(\blacksquare\)

Claim 3: The radical axis of \(\Gamma\) and \(\omega_A\) is \(\overline{EF}\).

Proof. Let \(S=\overline{BC}\cap\overline{EF}\). Note that \(S\) is the radical center of \(\Gamma\), \((MDX^*Y^*)\), and the nine-point circle. Moreover, the radical axis of \(\Gamma\) and \((MDX^*Y^*)\) must be perpendicular to \(\overline{VO}\), so it is \(\overline{EF}\). \(\blacksquare\)

Claim 4: The radical axis of \(\omega_B\), \(\omega_C\) is \(\overline{DT_A}\).

Proof. Let \(U_B=\overline{H_AH_A}\cap\overline{CC}\in\omega_B\) and \(U_C=\overline{H_AH_A}\cap\overline{BB}\in\omega_C\) by Claim 1. Since \(T_BT_CU_BU_C\) is tangential and \(\overline{AH_A}\perp\overline{BC}\), it is bicentric (e.g.\ by angle-chasing).

[asy]             size(7cm); defaultpen(fontsize(10pt)); pen pri=blue; pen pri1=lightblue; pen pri2=lightblue+white; pen sec=fuchsia; pen tri=lightred; pen qua=purple+pink; pen qui=heavycyan; pen fil=invisible; pen sfil=invisible; pen tfil=invisible; pen qfil=invisible;

pair O,A,B,C,D,EE,F,H,TA,TB,TC,NN,HA,HB,HC,X,Y,Z,K,Z,Bp,Cp,L,Xs,Ys,M,P,UB,UC,VB,VC; O=origin; A=dir(70+200); B=dir(210+200); C=dir(330+200); D=foot(A,B,C); EE=foot(B,C,A); F=foot(C,A,B); H=A+B+C; TA=2/(1/B+1/C); TB=2/(1/C+1/A); TC=2/(1/A+1/B); NN=H/2; HA=-B*C/A; HB=-A*C/B; HC=-A*B/C; X=extension(TA,B,H,H+rotate(90)*(A-NN)); Y=extension(TA,C,H,H+rotate(90)*(A-NN)); K=2A-H; Z=extension(B,HC,C,HB); Bp=-B; Cp=-C; L=(A+K)/2; Xs=(B+HC)/2; Ys=(C+HB)/2; M=(B+C)/2; P=extension(EE,F,B,TA); UB=2/(1/C+1/HA); UC=2/(1/B+1/HA); VB=2/(1/A+1/HC); VC=2/(1/A+1/HB);

filldraw(circumcircle(TB,UB,VB),qfil,qua); filldraw(circumcircle(TC,UC,VC),qfil,qua); draw(TA--(intersectionpoints(circumcircle(TB,UB,VB),circumcircle(TC,UC,VC))[1]+0.2*(D-TA)),tri+dashed); draw(UB--UC,sec); draw(B--HA--C,pri1); draw(A--HA,pri1); filldraw(TA--TB--TC--cycle,sfil,sec); filldraw(unitcircle,fil,pri); filldraw(A--B--C--cycle,fil,pri);

dot("\(A\)",A,A); dot("\(B\)",B,B); dot("\(C\)",C,C); dot("\(T_A\)",TA,N); dot("\(T_B\)",TB,SW); dot("\(T_C\)",TC,SE); dot("\(H_A\)",HA,dir(120)); dot("\(D\)",D,dir(-20)); dot("\(U_B\)",UB,NW); dot("\(U_C\)",UC,1.5N);         [/asy]

Then note that \(D\) is the radical center of \(\omega_B\), \(\omega_C\), \(\Gamma\) (by Claim 3), and from the above, \(T_A\) is the radical center of \(\omega_B\), \(\omega_C\), and the circle through \(T_B\), \(T_C\), \(U_B\), \(U_C\), so the claim holds. \(\blacksquare\)

Finally, recall \(\overline{EF}\parallel\overline{T_BT_C}\), and that \(H\) is the incenter of \(\triangle DEF\). Thus \(\triangle T_AT_BT_C\cup O\) and \(\triangle DEF\cup H\) are homothetic, so the four lines \(\overline{DT_A}\), \(\overline{ET_B}\), \(\overline{FT_C}\), \(\overline{OH}\) concur at the center of homothety.
This post has been edited 1 time. Last edited by TheUltimate123, Feb 22, 2021, 8:07 AM
Z K Y
The post below has been deleted. Click to close.
This post has been deleted. Click here to see post.
KST2003
173 posts
#36 • 2 Y
Y by icematrix2, GeoKing
Oh man this is a Hard problem.
First we will define a whole bunch of new points. Let $\triangle DEF$ and $\triangle T_aT_bT_c$ be the orthic triangle and tangential triangle respectively. Let $O$ and $O_T$ be the circumcenters of $\triangle ABC$ and $\triangle T_aT_bT_c$ respectively. Let $P$ and $Q$ be the foots of internal angle bisectors of $\angle T_c$ and $\angle T_b$ respectively, and let $I_A,I_B$ and $I_C$ be the corresponding excenters of $\triangle T_aT_bT_c$. Finally, let $A_1$ and $A_2$ be the intersections of $EF$ with $(ABC)$.
Notice that $\measuredangle EFA=\measuredangle ECB=\measuredangle T_cAF$, so $EF\parallel T_bT_c$, and consequently $\triangle DEF$ is homothetic to $\triangle T_aT_bT_c$. Now define points $X$ and $Y$ on segments $T_bC$ and $T_cB$, such that $\frac{CX}{CT_b}=\frac{DH}{DA}=\frac{BY}{BT_c}$, and let $Z$ be the inverse of $D$ with respect to $(ABC)$.

Claim 1: $X$ and $Y$ lie on $\omega_A$.

Proof. It is well known that $Z$ is the second intersection of $(T_aBC)$ and $(T_aT_bT_c)$, and that $ZA$ bisects $\angle T_bZT_c$. Therefore, we have
\[\frac{CD}{DB}=\frac{CZ}{ZB}=\frac{T_bZ}{ZT_c}=\frac{T_bA}{AT_c}\]which shows that $X,H,Y$ are collinear. Now, as $\triangle DEF$ and $\triangle T_aT_bT_c$ are homothetic, we see that $\triangle ABC$ is homothetic with $\triangle I_AI_BI_C$, so $AN$ is parallel to $I_AO_T$. Now it is well known that $I_AO_T$ is perpendicular to $PQ$, so we just have to show that $XY$ is parallel to $PQ$ to prove our claim. This is done by length bashing. Let $a,b,c$ denote the lengths of segments $T_bT_c,T_cT_a$ and $T_aT_b$. First, by angle bisector theorem, we have
\[T_aP=\frac{bc}{a+b}\qquad\text{and}\qquad T_aQ=\frac{bc}{a+c}.\]Therefore, their ratio is $\frac{T_aP}{T_aQ}=\frac{a+c}{a+b}$. Now by homothety, we have $\frac{DH}{DA}=\frac{T_aO}{T_aI_A}=\frac{s-a}{s}$, so
\[CX=\frac{(s-a)(s-b)}{s}\qquad\text{and}\qquad\frac{(s-a)(s-c)}{s}.\]Now computing $T_aX$ and $T_aY$ shows that
\[T_aX=(s-a)\left(\frac{s-b}{s}+1\right)=\frac{(s-a)(a+c)}{s}\qquad\text{and}\qquad T_aY=\frac{(s-a)(a+b)}{s}\]which means that $\frac{T_aX}{T_aY}=\frac{T_aP}{T_aQ}$ so $PQ\parallel XY$ as desired. In particular, since $\frac{CX}{XT_b}=\frac{BY}{YT_c}$, we have also shown that $\omega_A$ passes through $Z$. $\blacksquare$

Let $O_A$ be the center of $\omega_A$. Then by spiral similarity, we see that $\omega_A$ lies on the line joining the circumcenter of $(T_aBC)$, say $R$, and $O$. Moreover,
\[\frac{RO_A}{RO_T}=\frac{AO}{AI_A}=\frac{RO}{RS}\]where $S$ is the midpoint of arc $T_bT_c$. Therefore, $OO_A$ is perpendicular to $T_bT_c$, which means that $O_A$ lies on $AO$.

Claim 2: $A_1$ and $A_2$ also lie on $\omega$.

Proof. Let $EF$ meet $BC$ at $K$. Then as $(B,C;D,K)=-1$, it follows that
\[KA_1\cdot KA_2=KB\cdot KC=KD\cdot KM\]where $M$ is the midpoint of $BC$. Inverting with respect to $(ABC)$ sends $D$ to $Z$ and $M$ to $T_a$, so this means that $T_a,A_1,A_2,Z$ are concyclic. Notice that the perpendicular bisector of $A_1A_2$ coincides with $OA$. This forces $(T_aZA_1A_2)$ to be $\omega_A$. $\blacksquare$

Let $X'$ be the reflection of $X$ over $C$, and let $Y'$ be the reflection of $Y$ over $B$. Since the length of $CX$ was symmetric in $a$ and $b$, we see that $X'$ is the intersection of $\omega_B$ with $T_aT_b$. Similarly, $Y'$ lies on $\omega_C$. Now we will make the final crucial claim.

Claim 3: $B,C,Y',X'$ are cyclic.

Proof. This is another easy length bash. We just need to show that
\[\frac{T_aX'}{T_aY'}=\frac{b}{c}.\]From the quantities in claim 1, we have
\[T_aX'=T_aC-CX'=(s-a)-\frac{(s-a)(s-b)}{s}=(s-a)\left(1-\frac{s-b}{s}\right)=\frac{b(s-a)}{s}.\]Similarly, we have $AY'=\frac{c(s-a)}{s}$ so we're done. $\blacksquare$
We are really close to finishing now. Since by radical axis on $(T_bT_cY'X')$, $\omega_B$ and $\omega_C$, we see that $T_a$ lies on the radical axis of $\omega_B$ and $\omega_C$. Also, by radical axis on $(ABC),\omega_B$ and $\omega_C$, we see that $D$ also lies on this radical axis too. Hence the radical center of $\omega_A,\omega_B$ and $\omega_C$ is the concurrency point of $T_aD$, $T_bE$ and $T_cF$, which is the center of homothety which maps $\triangle A_1B_1C_1$ to $\triangle DEF$. In particular, this maps $O$ to $H$, so the concurrency point lies on the Euler line indeed.
This post has been edited 1 time. Last edited by KST2003, Apr 4, 2021, 6:26 PM
Z K Y
The post below has been deleted. Click to close.
This post has been deleted. Click here to see post.
rafaello
1079 posts
#39 • 1 Y
Y by geometry6
Let $PQR$ be the triangle, such that $(ABC)$ is tangent to $PQ,QR,RP$ at $C,A,B$, respectively. Let $H_A$ be the reflection of $H$ over $BC$, $H_B$ be the reflection of $H$ over $AC$ and $H_C$ be the reflection of $H$ over $AB$. Let $DEFGIJ$ be the hexagon, such that $(ABC)$ is tangent to $DE,EF,FG,GI,IJ,JD$ at $H_A,C,H_B,A,H_C,B$, respectively.

[asy]import geometry;import olympiad;
size(12cm); defaultpen(fontsize(10pt));pen org=magenta;pen med=mediummagenta;pen light=pink;pen deep=deepmagenta;pen dark=darkmagenta;
pair O,A,B,C,H,h,N,D,E,F,G,K,g,j,I,J,P,Q,R,Y,X,T,U,V;
O=(0,0);A=dir(120);B=dir(215);C=dir(325);H=orthocenter(A,B,C);h=2foot(A,B,C)-H;g=2foot(B,A,C)-H;j=2foot(C,A,B)-H;N=midpoint(O--H);D=intersectionpoint(perpendicular(B,line(B,O)),perpendicular(h,line(h,O)));
E=intersectionpoint(perpendicular(C,line(C,O)),perpendicular(h,line(h,O)));
G=intersectionpoint(perpendicular(C,line(C,O)),perpendicular(g,line(g,O)));
K=intersectionpoint(perpendicular(A,line(A,O)),perpendicular(g,line(g,O)));
I=intersectionpoint(perpendicular(A,line(A,O)),perpendicular(j,line(j,O)));
J=intersectionpoint(perpendicular(B,line(B,O)),perpendicular(j,line(j,O)));
P=extension(D,B,C,E);
Q=extension(C,G,A,K);
R=extension(A,I,J,B);
Y=extension(B,C,R,E);
X=extension(P,Y,O,H);
T=extension(B,C,A,h);
V=extension(X,R,A,B);
U=extension(A,C,X,Q);



draw(A--B--C--cycle,deep);draw(circumcircle(A,B,C),deep);draw(P--Q--R--cycle,deep);draw(J--I--K--G--E--D--cycle,org);draw(R--E,light);draw(D--Q,light);draw(A--h,light);draw(R--X--P,dark);draw(X--Q,dark);
draw(T--U--V--cycle,dark);draw(X--O,dark+dashed);

dot("$O$",O,dir(0));
dot("$A$",A,dir(A));
dot("$B$",B,dir(B));
dot("$C$",C,dir(C));
dot("$H_A$",h,dir(h));
dot("$D$",D,dir(D));
dot("$E$",E,dir(E));
dot("$H_B$",g,dir(g));
dot("$H_C$",j,dir(j));
dot("$F$",G,dir(G));
dot("$H$",H,dir(0));
dot("$I$",I,dir(I));
dot("$J$",J,dir(J));dot("$G$",K,dir(K));dot("$R$",R,dir(R));dot("$Q$",Q,dir(Q));dot("$P$",P,dir(P));dot("$X$",X,dir(X));dot("$T$",T,dir(T));dot("$U$",U,dir(U));dot("$V$",V,dir(V));
[/asy]



Claim. Let $H,O,N$ be the orthocentre, circumcentre and centre of nine-point circle of triangle $ABC$. Let $H_A$ be the reflection of $H$ over $BC$. Let $D$ be the intersection of tangents from $B,H_A$ to $(ABC)$. Then, $DH\perp CN$.
Proof.
[asy]import geometry;import olympiad;
size(5cm); defaultpen(fontsize(10pt));pen org=magenta;pen med=mediummagenta;pen light=pink;pen deep=deepmagenta;
pair O,A,B,C,H,h,N,D,P,L;
O=(0,0);A=dir(105);B=dir(195);C=dir(345);H=orthocenter(A,B,C);h=2foot(A,B,C)-H;N=midpoint(O--H);D=intersectionpoint(perpendicular(B,line(B,O)),perpendicular(h,line(h,O)));P=extension(C,N,H,D);L=2P-H;

draw(A--B--C--cycle,deep);draw(circumcircle(A,B,C),deep);draw(B--D--h,magenta);draw(H--D,deep);draw(C--P,light);draw(C--H,light);draw(C--h,light);draw(circumcircle(O,B,D),magenta+dashed);draw(O--H,deep);draw(circumcircle(H,L,h),light);
draw(L--O,magenta);draw(C--L,light);draw(O--h,magenta);draw(O--B,magenta);

clip((1.05,1.05)--(-1.05,1.05)--(-1.05,-1.05)--(1.05,-1.05)--cycle);

dot("$O$",O,dir(90));
dot("$A$",A,dir(A));
dot("$B$",B,dir(B));
dot("$C$",C,dir(C));
dot("$H$",H,dir(H));
dot("$H_A$",h,dir(h));
dot("$N$",N,dir(N));
dot("$D$",D,dir(D));
dot("$P$",P,dir(P));
dot("$L$",L,dir(L));
[/asy]

Let $L$ be the point on $DH$ such that $CH=CH_A=CL$. Then,
$$\measuredangle DLH_A=\measuredangle HLH_A=\frac{\measuredangle HCH_A}{2} =\measuredangle BCH_A=\measuredangle DBH_A,$$hence $L$ lies on $(OH_ADB)$. Thus, $LO\perp DH$. Let $P$ be the midpoint of $HL$. Hence, $PN\perp DH\perp PC$, hence $P,N,C$ are collinear, we conclude that $CN\perp DH$.

Also,
\begin{align*}
\measuredangle EDR=\measuredangle H_ADB=\measuredangle H_AOB=2\measuredangle H_AAB\\=2\measuredangle CAO=\measuredangle COA=\measuredangle CQA=\measuredangle EQR,
\end{align*}hence $DEQR$ is cyclic, similarly $IJPQ$ and $GFPR$ is cyclic. Now, by radical axis, $P$ lies on the common chord of $\omega_B,\omega_C$. Similarly, $Q$ lies on the common chord of $\omega_A,\omega_C$ and $R$ lies on the common chord of $\omega_A,\omega_B$.


By the Brainchon's theorem, we also get that $AH_A,BC,ER,DQ$ are concurrent, call that concurrency point $T$. Similarly define $U$ and $V$.

Now comes the hard part, we would like to show that $T$ lies on the radical axis of $\omega_A,\omega_B$ and so on.
Claim. $T$ lies on the radical axis of $\omega_A,\omega_B$.
Proof.
Firstly we do inversion wrt $(ABC)$. Then bunch of intersections of tangent lines map to the midpoints of chords, which is cute (for the labeling, refer to the diagram below, in short $X'$ be the image of $X$ wrt $(ABC)$. Also let $K$ be the midpoint of $CH$, $L$ be the midpoint of $AH$, $M$ be the midpoint of $BH$ $Z$ the reflection of $T$ over $AB$ and $W$ the reflection of $V$ over $BC$. Note that by PoP, $I'ZUQ'$ and $WE'UQ'$ are cyclic and we easily get that $I'ZE'W$ is cyclic. Now by radical axis, $I'UQ'E'$ is cyclic. Similarly $G'VR'D'$ is cyclic.

By radical axis theorem on $\Gamma_1=(I'UQ'E'),\Gamma_2=(G'VR'D')$ and nine-point circle, we get that $A$ lies on the radical axis of $\Gamma_1,\Gamma_2$.
Let $K'$ be the reflection of $K$ over $C$, $M'$ be the reflection of $M$ over $B$. Note that $D',E',K'$ are collinear. Thus, by angle chase $K'$ lies on $\Gamma_2$. Similarly $M'$ lies on lies on $\Gamma_1$.
Thus, by PoP, $H$ lies on the radical axis of $\Gamma_1,\Gamma_2$. Hence $T$ also lies on the radical axis of $\Gamma_1,\Gamma_2$.

Now comes the only non-synthetic part :(, we use the linearity of PoP.
Let $f(\bullet)=P(\bullet,\Gamma_2)-P(\bullet,(ABC))$. We know that $f$ is linear.
Hence,
\begin{align*}
f(T)=\frac{CT}{BC}f(B)+\frac{BT}{BC}f(C)=\frac{CT}{BC}\left(BR'\cdot BV \right)+\frac{BT}{BC}\left(-CK'\cdot CV\right)\\=\frac{CT}{BC}\left(BT\cdot BP'\right)+\frac{BT}{BC}\left(-CP'\cdot CT\right)=0.
\end{align*}
By radical axis theorem on $(BOC),\Gamma_2$ and $(ABC)$, $T$ lies on the radical axis of $(BOC)$ and $\Gamma_2$. Now radical axis on $(BOC),\Gamma_2$ and circle formed by $O$ and intersection points of $\Gamma_1,\Gamma_2$, call this circle $\Omega$, we get that $T$ lies on $\Omega,(BOC)$. As $T'$ lies on $OT$ and $T'$ lies on $(BOC)$, we get that $T'$ lies on $\Omega$.
We invert back and $T$ lies on the radical axis of $\omega_A,\omega_B$.
[asy]
size(12cm); defaultpen(fontsize(10pt));pen org=magenta;pen med=mediummagenta;pen light=pink;pen deep=deepmagenta;
pair O,A,B,C,H,Ha,Hb,Hc,T,D,E,F,G,I,J,U,V,K,k,P,Q,R,L,W,Z,M,m,t,i;
O=(0,0);A=dir(105);B=dir(195);C=dir(345);path w=circumcircle(A,B,C);H=orthocenter(A,B,C);Ha=2*foot(A,B,C)-H;Hb=2*foot(B,A,C)-H;Hc=2*foot(C,A,B)-H;T=foot(A,B,C);D=midpoint(B--Ha);E=midpoint(C--Ha);F=midpoint(C--Hb);G=midpoint(Hb--A);I=midpoint(Hc--A);J=midpoint(Hc--B);U=foot(B,A,C);V=foot(C,B,A);K=midpoint(C--H);
path w1=circumcircle(I,U,E);path w2=circumcircle(V,G,D);k=2*C-K;P=midpoint(B--C);Q=midpoint(A--C);R=midpoint(A--B);L=midpoint(A--H);W=2foot(V,B,C)-V;Z=2foot(T,B,A)-T;M=midpoint(B--H);m=2B-M;t=intersectionpoints(circumcircle(B,O,C),circumcircle(A,O,Ha))[1];i=intersectionpoints(circumcircle(U,Q,I),circumcircle(G,V,R))[0];

draw(A--B--C--cycle,deep);draw(w,deep);draw(B--Ha--C--Hb--A--Hc--cycle,org);draw(A--T,org);draw(B--V,org);draw(C--U,org);draw(w1,med);draw(w2,med);draw(C--V,med);draw(B--U,med);draw(circumcircle(T,U,V),med+dashed);draw(A--Z,org);draw(C--W,org);draw(circumcircle(B,O,C),org);
draw(circumcircle(i,P,t),deep+dashed);draw(circumcircle(A,Ha,t),org);

clip((2,2)--(2,-2)--(-2,-2)--(-2,2)--cycle);


dot("$O$",O,dir(90));dot("$A$",A,dir(A));dot("$B$",B,dir(B));dot("$C$",C,dir(C));dot("$H$",H,dir(H));dot("$H_A$",Ha,dir(Ha));dot("$H_B$",Hb,dir(Hb));dot("$H_C$",Hc,dir(Hc));dot("$T$",T,dir(T));dot("$D'$",D,dir(D));dot("$E'$",E,dir(E));dot("$F'$",F,dir(F));dot("$G'$",G,dir(G));dot("$J'$",J,dir(J));dot("$I'$",I,dir(I));dot("$U$",U,dir(U));dot("$V$",V,dir(V));dot("$K$",K,dir(K));dot("$K'$",k,dir(k));dot("$P'$",P,dir(P));dot("$Q'$",Q,dir(Q));dot("$R'$",R,dir(R));dot("$L$",L,dir(L));dot("$W$",W,dir(W));dot("$Z$",Z,dir(Z));dot("$M$",M,dir(M));dot("$M'$",m,dir(m));dot("$T'$",t,dir(45));
[/asy]

By the claim, similarly $U$ lies on the radical axis of $\omega_A,\omega_C$ and $V$ lies on the radical axis of $\omega_B,\omega_C$.

Now, note that $RQ\parallel UV$ as $\measuredangle AUV=\measuredangle CBA=\measuredangle UAQ$. Similarly, we get that $PR\parallel VT$ and $PQ\parallel TU$, which means that $TUV$ and $PQR$ are homothetic. Now, note that $O$ is the incenter of $\triangle PQR$ and $H$ is the incenter of $\triangle TUV$. Hence, $PT,QU,RV,OH$ are concurrent.
Z K Y
The post below has been deleted. Click to close.
This post has been deleted. Click here to see post.
CT17
1481 posts
#40 • 1 Y
Y by GeoKing
Excellent problem. Solved with heavy use of Geogebra.

Let $T_AT_BT_C$ be the triangle determined by the tangents to $\Gamma$ at $A,B,C$. Let $Q_C\in T_AB$ and $P_B\in T_AC$ so that $T_BT_CQ_CP_B$ is cyclic and $\Gamma$ is an excircle of $\triangle T_AQ_CP_B$. Define $Q_A,P_C,Q_B,P_A$ analogously so that $Q_CP_BQ_AP_CQ_BP_A$ is a convex hexagon inscribed in $\triangle T_AT_BT_C$ with incircle $\Gamma$. The goal is to show that $\omega_A = (T_AP_AQ_A)$ and similar.

Claim 1: Let $\Gamma$ be tangent to $Q_CP_B$ at $X$. Then $AX$, $T_CP_B$, and $T_BQ_C$ concur at the foot $D$ of the altitude from $A$ to $BC$.

Proof: Follows from well-known properties of bicentric quadrilaterals.$\square$

Claim 2: Let $O'$ be the reflection of $O$ over $AC$. Then $DO'\perp DT_C$.

Proof (I'm sorry): Let $A=a, B=b, C=c$ on the complex unit circle. Then we have $D = \frac{a+b+c-\frac{bc}{a}}{2}$, $T_C = \frac{2ab}{a+b}$, and $O' = a+c$. Hence

$$\frac{T_C-D}{O'-D} = \frac{\frac{2ab}{a+b} - \frac{a+b+c-\frac{bc}{a}}{2}}{a+c-\frac{a+b+c-\frac{bc}{a}}{2}}=\frac{4a^2b-a^3-ab^2-2a^2b-a^2c-abc+abc+b^2c}{(a+b)(a^2+ac-ab+bc)} = \frac{-(a-b)(a^2+ac-ab+bc)}{(a+b)(a^2+ac-ab+bc)}$$
which is evidently purely imaginary (apparently as is my commitment to synthetic).$\square$

Claim 3: $HP_B\perp BN$.

Proof: Let $O'$ be the reflection of $O$ over $AC$ so that $N$ is the midpoint of $BO'$. Let $AO'\cap CT_B = Y$. As $O'$ is the orthocenter of $\triangle T_BAC$, we have $\angle AYC = 90^\circ$. In particular, if $K$ is the foot from $P_B$ to $BO'$, $ADCY$ and $O'DP_BY$ are cyclic with diameters $AC$ and $O'P_B$, so we have a spiral similarity $\triangle DAC\sim\triangle DO'P_B$. Now phantom $K'$ as the foot from $H$ to $AO'$. We have

$$\measuredangle DKB = \measuredangle DKO' = \measuredangle DP_BO' = \measuredangle DCA = \measuredangle DHB = \measuredangle DK'B$$
so $K = K'$, as desired.$\square$

From claim $3$ (and analogous results for the other five vertices of the hexagon) it follows that $\triangle T_AP_AQ_A$ etc. are indeed the triangles defined in the problem statement (with circumcircles $\omega_A$ etc.) From now on we will work with $\omega_B$ and $\omega_C$.

Claim 4: $T_A$ lies on the radical axis of $\omega_B$ and $\omega_C$.

Proof: As $T_BT_CQ_CP_B$ is cyclic, we have

$$\text{pow}_{\omega_C}(T_A) = T_AQ_C\cdot T_AT_C = T_AP_B\cdot T_AT_B = \text{pow}_{\omega_B}(T_A).\square$$
Claim 5: $Z = T_AD\cap T_BT_C$ lies on the radical axis of $\omega_B$ and $\omega_C$.

Proof: By linearity of power of a point, it suffices to show that $\frac{\text{pow}_{\omega_B}(T_C)}{\text{pow}_{\omega_C}(T_B)} = \frac{ZT_C}{ZT_B}$. Indeed, if $r_B$ and $r_C$ are the $T_B-$ and $T_C-$ exradii of $\triangle T_AT_BT_C$ and $r$ is the radius of $\Gamma$ we have

$$\frac{\text{pow}_{\omega_B}(T_C)}{\text{pow}_{\omega_C}(T_B)} = \frac{T_CQ_B}{T_BP_C} = \frac{T_AT_C\cdot \frac{r}{r_C}}{T_AT_B\cdot \frac{r}{r_B}} = \frac{T_AT_C}{T_AT_B}\cdot \frac{r_B}{r_C} = \frac{T_AT_C}{T_AT_B}\cdot \frac{AT_C}{AT_B} = \frac{T_AT_C}{T_AT_B}\cdot \frac{DB}{DC} = \frac{ZT_C}{ZT_B}$$
as desired.$\square$

Now by symmetry, it follows that the radical center of $\omega_A$, $\omega_B$, and $\omega_C$ is the center of homothety $S$ of $\triangle T_AT_BT_C$ and the orthic triangle $\triangle DEF$. But the circumcenter $N$ of $\triangle DEF$ lies on $OH$, and it is well known (i.e. by inversion about $\Gamma$) that the circumcenter of $\triangle T_AT_BT_C$ lies on $OH$, so $S$ also lies on $OH$, as desired.$\square$
This post has been edited 1 time. Last edited by CT17, Oct 30, 2023, 10:11 PM
Z K Y
The post below has been deleted. Click to close.
This post has been deleted. Click here to see post.
cursed_tangent1434
606 posts
#41 • 5 Y
Y by ihategeo_1969, Shreyasharma, GeoKing, anantmudgal09, Ushan_Pinto
Amazing problem, USEMO continues to be the stronghold of configurational geometry. I should have guessed the problem proposer.

Let $D$ , $E$ and $F$ denote the feet of the altitudes of $\triangle ABC$ and let $H_a$ , $H_b$ and $H_c$ denote the reflections of the orthocenter $H$ across the sides $BC$ , $AC$ and $AB$ respectively. We start off by proving the following very important result.

Claim : The tangents to $\Gamma$ at $H_a$ and $C$ intersect on the line through $H$ perpendicular to $\overline{BN}$.

Proof : Let $A'$ and $C'$ denote the $A-$antipode and $C-$antipode respectively in $(ABC)$. Let $R= \overline{AH_c} \cap \overline{CH_a}$ and $H'$ the reflection of $H$ across $B$. Now, note that since
\[BH'=BH=BH_a=BH_c\]$B$ is the center of the circle $(HH_aH'H_c)$ with diameter $HH'$. Thus, $\measuredangle H'H_aH = \measuredangle HH_cH' = \frac{\pi}{2}$ which implies that points $H'$ , $H_a$ and $A'$ (and points $H'$ , $H_c$ and $C'$) are collinear. By Pascal's Theorem on concyclic hexagon $AH_cC'CH_aA'$ it follows that points $R$ , $H'$ and $O$ are collinear. Thus, by the Midpoint Theorem it follows that $\overline{RO} \parallel BN$.

Note that by Brokard's Theorem, $H$ lies on the polar of $R$ with respect to $(ABC)$, and in particular the polar of $R$ with respect to $(ABC)$ is the line through $H$ perpendicular to $\overline{OR}$. But since we noted that $OR \parallel BN$, the polar of $R$ is simply the line through $H$ perpendicular to $\overline{BN}$. Let $P$ and $Q$ denote the intersections of the line through $H$ perpendicular to $\overline{BN}$ with $(ABC)$. Thus, $PH_aQC$ must be harmonic. From this it follows that the tangents to $\Gamma$ at $H_a$ and $C$ indeed intersect on the line through $H$ perpendicular to $\overline{BN}$, as claimed.

Now, let $T_a$ denote the intersection of the tangents to $\Gamma$ at $B$ and $C$. Define $T_b$ and $T_c$ similarly. Let $\ell_a$ denote the line through $H$ perpendicular to $\overline{AN}$. Define $\ell_b$ and $\ell_c$ similarly. Let $X_b$ and $X_c$ denote the intersections of $\ell_a$ with the tangents to $\Gamma$ at $B$ and $C$ respectively. Define the points $Y_a$ , $Y_c$ and $Z_a$ , $Z_b$ similarly.

As a result of the previous claim, it follows that $Y_c$ and $Z_b$ lie on the tangent to $\Gamma$ at $H_a$ (and similarly). Now, we can make the following key observation.

Claim : Quadrilateral $T_cZ_bY_cT_c$ (and similar) is cyclic.

Proof : This is a direct angle chase. Note that,
\[\measuredangle Z_bY_cT_c = \measuredangle H_aY_cC = 2 \measuredangle H_aCY_c = 2\measuredangle H_aAC = 2\measuredangle BCA = 2\measuredangle T_cBA = \measuredangle BT_cA = \measuredangle Z_bT_cT_b\]which implies the claim.

Now, note that,
\[\text{Pow}_{\omega_C}(T_a)=T_aZ_b \cdot T_aT_c = T_aY_c \cdot T_aT_b = \text{Pow}_{\omega_B}\]Thus, $T_a$ lies on the common chord of $\omega_B$ and $\omega_C$.

Claim : Line $\overline{DE}$ is the radical axis of circles $\Gamma$ and $\omega_C$ (and similarly).

Proof : Let $K= \overline{DE} \cap \overline{T_bT_c}$ and $L= \overline{DE} \cap \overline{T_aT_c}$. We show that $K$ lies on this radical axis, the proof for $L$ is entirely similar. Note that by Brianchon's Theorem on tangential quadrilateral $T_aX_cZ_aT_c$ lines $\overline{AC}$ , $\overline{BH_b}$ , $\overline{T_aX_c}$ and $\overline{T_cZ_a}$ concur. The former two clearly intersect at $E$ thus points $T_a$ , $E$ , $X_c$ and $T_c$ , $E$ and $Z_a$ are collinear. Now since $DE \parallel T_aT_b$ , we have that $\triangle KAE \sim \triangle T_bAC$ and thus $KA=KE$. Further,
\[\measuredangle Z_aEK = \measuredangle Z_aT_aX_c = \measuredangle Z_aT_cE\]so $\overline{KE}$ is tangent to $(Z_aT_cE)$. Thus,
\[KT_c \cdot KZ_a = KE^2 = KA^2\]which implies the claim.

Now, note that by the Radical Center Theorem on circles $\Gamma$ , $\omega_B$ and $\omega_C$, the radical axis of $\omega_B$ and $\omega_C$ , $\overline{DF}$ and $\overline{DE}$ must concur. The latter two intersect at $D$ implying that $D$ also lies on the common chord of circles $\omega_B$ and $\omega_C$. Thus, $\overline{DT_a}$ is the common chord of circles $\omega_B$ and $\omega_C$.

Similarly, it follows that $\overline{ET_B}$ and $\overline{FT_C}$ are the common chords of $\omega_A$ and $\omega_C$ and $\omega_A$ and $\omega_B$ respectively. Now, note that since $\triangle T_aT_bT_c$ and $\triangle DEF$ are homothetic, lines $\overline{DT_a}$ , $\overline{ET_b}$ and $\overline{FT_c}$ concur are the point $X_{25}$ - the homothetic center of the orthic and tangential triangles. Further since the incenter of $\triangle DEF$ is $H$ and the incenter of $\triangle T_aT_bT_c$ is $O$ is follows by homothety that $X_{25}$ lies on line $OH$, which solves the problem.
This post has been edited 1 time. Last edited by cursed_tangent1434, Dec 26, 2024, 7:49 AM
Reason: correction of proof of claim
Z K Y
The post below has been deleted. Click to close.
This post has been deleted. Click here to see post.
ihategeo_1969
218 posts
#42 • 1 Y
Y by cursed_tangent1434
My username is so accurate after this problem.

We define some new objects.
$\bullet$ Let $\triangle X_AX_BX_C$ be the tangential triangle of $\triangle ABC$.
$\bullet$ Let $\triangle DEF$ be the orthic triangle of $\triangle ABC$.
$\bullet$ Let $\ell_a$ be the line though $H$ perpendicular to $\overline{AN}$. Define $\ell_b$ and $\ell_c$ similarly.
$\bullet$ Let $A_B=\ell_a \cap \overline{BB}$. Define $A_C$, $B_A$, $B_C$, $C_A$, $C_B$ similarly.
$\bullet$ Let $H_A=\overline{AH} \cap \Gamma$, and define $H_B$, $H_C$ similarly.

Claim: $C_B$ is the pole of $\overline{BH_A}$ w.r.t. $\Gamma$ (and analogous).
Proof: Set $\Gamma$ to be the unit circle in complex plane. So we have \[h_a=b+c-bc\overline{h}=b+c-\frac{ab+bc+ca}a=-\frac{bc}a\]So let $C_B'$ be pole of $\overline{H_AB}$. So we have \[c_b'=\frac{2 \left(-\frac{bc}a \right)b}{b-\frac{bc}a}=\frac{2bc}{c-a}\]So we just need to prove $\overline{C_B'H} \perp \overline{CN}$ where $n=\frac{a+b+c}2$. And so we need to prove \[\frac{a+b+c-\frac{2bc}{c-a}}{c-\frac{a+b+c}2}=\frac{2(a+b+c)(c-a)-4bc}{(c-a)(c-a-b)}=\frac{2(c+a)(c-a-b)}{(c-a)(c-a-b)} =-\frac{2(\overline{c}+\overline{a})}{\overline{c}-\overline{a}}\]Hence the quantity is imaginary and we are done.. $\square$

Claim:$D=\overline{X_CB_C} \cap \overline{X_BC_B}$ (and analogous).
Proof: Apply Brianchon's theorem on $X_CBC_BB_CCX_B$ and $X_CAX_BB_CH_AC_B$. $\square$

Claim: $(X_CC_BB_CX_B)$ is cyclic (and analogous).
Proof: Simple angle chase \begin{align*} 
\angle X_CC_BB_C=\angle BC_BH_A &= 180 ^{\circ}-\angle BOH_A \\
&= 180 ^{\circ}-2 \angle BAH_A \\
&= 2 \angle ABC=\angle AOC \\
&= 180 ^{\circ}-\angle AX_BC \\
&= 180 ^{\circ}-\angle X_CX_BB_C 
\end{align*}And done. $\square$

Claim: $X_A$ lies on radical axis of $\omega_B$ and $\omega_C$ (and analogous).
Proof: Apply radical axis theorem on $(X_CC_BB_CX_B)$; $\omega_B$; $\omega_C$. $\square$

Claim:} $\overline{EF}$ is tangent to $(A_BFX_A)$ (and analogous).
Proof: See that $(A_BX_AX_BB_A)$ is cyclic and $F=\overline{A_BX_B} \cap \overline{X_AB_A}$. And hence \[\angle EFX_A=180^{\circ}-\angle EFB_A=\angle X_BB_AX_A=\angle FA_BX_A\]And so done. $\square$

Let $K_{A_C}=\overline{EF} \cap \overline{X_AX_C}$. Define $K_{A_B}$, $K_{B_A}$, $K_{B_C}$, $K_{C_A}$, $K_{C_B}$ similarly.

Claim: $K_{A_C}B=K_{A_C}F$ (and analogous)
Proof: See that $\overline{K_{A_C}F} \parallel \overline{AX_C}$ so homothety at $B$ finishes. $\square$

Claim: $\overline{FE}$ is the radical axis of $\Gamma$ and $\omega_A$ (and analogous).
Proof: We just need to prove $K_{A_C}$ has same power to both circles (and do the same for $K_{A_B}$). This is due to \[\text{Pow}_{\omega_A}\left(K_{A_C} \right)=K_{A_C}A_B \cdot K_{A_C}X_A=K_{A_C}F^2=K_{A_C}B^2=\text{Pow}_{\Gamma}\left(K_{A_C} \right)\]And done. $\square$

See that $D$ is radical center of $\Gamma$, $\omega_B$, $\omega_C$. So we get our main lemma.

Main Lemma: $\overline{X_AD}$ is the radical axis of $\omega_B$ and $\omega_C$ (and analogous).

Now $\triangle DEF$ is homothetic to $\triangle X_AX_BX_C$ and hence the center of homothety (i.e. radical center of $\omega_A$, $\omega_B$, $\omega_C$) lies on the line joining their incenters which is exactly $\overline{OH}$.

Remark: ``I am on a highway to hell".
Attachments:
This post has been edited 3 times. Last edited by ihategeo_1969, Jan 4, 2025, 11:49 AM
Z K Y
N Quick Reply
G
H
=
a